SlideShare uma empresa Scribd logo
1 de 140
Baixar para ler offline
Apostila de Matemática
Professor: Linhares e Júlio.

 [...] a Matemática procura compreender os modelos
                que permeiam o mundo que
  nos rodeia assim como a mente dentro de nós. […]
               Assim é necessário enfatizar:
        — a procurar de soluções, e não apenas a
              memorização de procedimentos;
       — a exploração de modelos, e não apenas a
                memorização de fórmulas;
    — a formulação de conjecturas, e não apenas a
                  resolução de exercícios.
      [...] com essas ênfases, os estudantes terão a
          oportunidade de estudar a Matemática
 como uma disciplina exploradora, dinâmica, que se
                    desenvolve, em lugar
     de ser uma disciplina que tem um corpo rígido,
             absoluto, fechado, cheio de regras
              que precisam ser memorizadas.
                     Shoenfeld (1992).


26/01/2011
Análise Combinatória
Fatorial de um número:

  n!=n.(n-1).(n-2)...3.2.1

Definições especiais: 0!=1
                      1!=1


                            100!101!
1) Calcule o valor da expressão       .
                               99!
100!101! 100.99!101.100.99!
                              100  101.100  100  10100  10200
   99!            99!

                      ( x  1)!
2) Resolva a equação              56.
                      ( x  1)!
( x  1)!        ( x  1)( x)( x  1)!
           56                         56  ( x  1)( x)  56  x 2  x  56 
( x  1)!              ( x  1)!
                                  1  225            1  15     x  7
   x 2  x  56  0  x                       x            
                                      2                 2         x  -8
Resposta : x  7, pois não existe fatorial de um número negativo.

3) Quatro times de futebol (Grêmio, Santos, São Paulo e Flamengo) disputam o torneio dos
campeões do mundo. Quantas são as possibilidades para os três primeiros lugares?
R : Existem 4 possibilidades para o 1º lugar, sobrando 3 possibilidades para o 2º lugar e 2
possibilidades para o 3º lugar  4.3.2  24 possibilidades.

Arranjo simples:

                       n!
        An, p 
                    (n  p)!


               A6, 2  A4,3  A5, 2
4) Calcule                            .
                    A9, 2  A8,1
                             6!         4!        5!
                                           
 A6, 2  A4,3  A5, 2     (6  2)! (4  3)! (5  2)! 30  24  20 34 17
                                                                  
     A9, 2  A8,1                  9!       8!          72  8     80 40
                                        
                                (9  2)! (8  1)!

Prof: Linhares e Júlio                                                              Página 2
5) Quantos números de 3 algarismos distintos podemos formar com o algarismos do
sistema decimal (0,1,2,3,4,5,6,7,8,9) sem os repetir, de modo que :
a) COM ECEM COM 1.
   R : O número pode possuir três algarismos , sendo que para o primeiro existe apenas 1
possibilidade (1) e para os outros dois ainda existem 9 números disponíveis :
                   9!    9! 9.8.7!
   1. A9, 2                      9.8  72 números.
                (9  2)! 7!   7!

b) COM ECEM COM 2 E TERM INEM COM 5.
   R : Para o primeiro algarismo existe apenas 1 possibilidade (2), e para o terceiro também
existe apenas 1 possibilidade (5). Para o segundo ainda existem 8 possibilidades :
                 8!    8! 8.7!
  1.1. A8,1                    8 números.
              (8  1)! 7! 7!


c) SEJAM DIVISÍVEIS POR 5.
   R : Para um número ser divisível 5, ele deve terminar com 0 ou com 5. Primeirame nte
vamos calcular o número de divisíveis por 5 que terminam com 0 :
 Para o terceiro algarismo existe apenas 1 possibilidade (0), e para os dois primeiros ainda
existem 9 números disponíveis. Portanto o número de divisíveis por 5 que terminam com 0 é :
                9!    9! 9.8.7!
  1. A9, 2                    9.8  72 números.
             (9  2)! 7!   7!
 Agora calculamos quantos divisíveis por 5 terminam com 5 : para o terceiro algarismo
existe apenas uma possibilidade (5). Para o primeiro algarismo existem ainda 8 possibilidades,
pois o número não pode começar com 0 (senão seria um número de 2 algarismos ). E para o
segundo algarismo também existem 8 possibilidades (o segundo algarismo pode ser 0).
                      8!       8!    8! 8! 8.7! 8.7!
  1. A8,1 . A8,1          .         .       .      8.8  64 números.
                   (8  1)! (8  1)! 7! 7! 7! 7!
Resposta : O número de divisíveis por 5 é 72  64  136 números.


6) Quantos são os números compreendidos entre 2000 e 3000 formados por algarismos
distintos escolhidos entre 1,2,3,4,5,6,7,8 e 9?
  R : O número deve ter quatro algarismos (pois está entre 2000 e 3000). Para o primeiro
algarismo existe apenas uma possibilidade (2), e para os outros três ainda existem 8 números
disponíveis, então :
                   8!    8! 8.7.6.5!
   1. A8,3                         8.7.6  336 números.
                (8  3)! 5!    5!



Prof: Linhares e Júlio                                                             Página 3
Permutação Simples: É um caso particular de arranjo simples. É o tipo de
agrupamento ordenado onde entram todos os elementos.


                                 Pn  n!

7) Quantos números de 5 algarismos distintos podem ser formados por 1,2,3,5 e 8?
P5  5! 5.4.3.2.1  120 números.


8) Quantos anagramas da palavra EDITORA :
a) COM EÇAM POR A.
   Para a primeira letra existe apenas uma possibilidade (A), e para as outras 6 letras
existem 6 possibilidades. Então o total é :
1.P6  1.6! 6.5.4.3.2.1  720 anagramas.


b) COM EÇAM POR A e terminam com E.
   Para a primeira letra existe 1 possibilidade (A), e para última também só existe 1 (E),
e para as outras 5 letras existem 5 possibilidades. Então o total é :
1.1.P5  1.1.5! 5.4.3.2.1  120 anagramas.


8) Calcule de quantas maneiras podem ser dipostas 4 damas e 4 cavalheiro s, numa fila, de
forma que não fiquem juntos dois cavalheiro s e duas damas.
   R :Existem duas maneiras de fazer isso :
     C - D - C - D - C - D - C - D ou D - C - D - C - D - C - D - C
Colocando um cavalheiro na primeira posição temos como número total de maneiras :
P4 .P4  4!.4! 24.24  576 maneiras.
Colocando uma dama na primeira posição temos também :
P4 .P4  4!.4! 24.24  576 maneiras.
Portanto o total é 576  576  1152 maneiras.


Combinação Simples: é o tipo de agrupamento em que um grupo difere do
outro apenas pela natureza dos elementos componentes.


                                            n!
                            Cn, p 
                                        p!(n  p)!
Prof: Linhares e Júlio                                                               Página 4
9) Resolver a equação C m ,3  C m , 2  0.
    m!           m!
                       0
3!(m  3)! 2!(m  2)!
m.(m  1).(m  2).(m  3)! m.(m  1).(m  2)!
                                                 0
        3!(m  3)!                2!(m  2)!
m.(m  1).(m  2) m.(m  1)
                               0
        3!               2!
m 3  2m 2  m 2  2m m 2  m
                                 0
           6                 2
m 3  3m 2  2m  3m 2  3m
                                0  m 3  6 m 2  5m  0
              6
                            6  16        m '  5
m 2  6m  5  0  m                  
                                2         m ' '  1
Resposta : m  5.
obs : m  1 não é a resposta porque não pode haver C1,3.


10) Com 10 espécies de frutas, quantos tipos de salada, contendo 6 espécies diferentes
podem ser feitas?
                10!       10.9.8.7.6! 5040 5040
C10, 6                                       210 tipos de saladas.
           6!.(10  6)!      6!.4!     4!   24

11) Numa reunião com 7 rapazes e 6 moças, quantas comissões podemos formar com 3
rapazes e 4 moças?
RAPAZES- C 7 ,3
M OÇAS- C 6, 4
O resultado é o produto C 7 ,3 .C 6, 4 .
    7!         6!       7.6.5.4! 6.5.4! 210 30
          .                    .         .  35.15  525 comissões.
3!(7  3)! 4!(6  4)!     3!.4! 4!.2!    3! 2




Prof: Linhares e Júlio                                                             Página 5
Binômio de Newton
Introdução
    Pelos produtos notáveis, sabemos que (a+b)² = a² + 2ab + b².
   Se quisermos calcular (a + b)³, podemos escrever:

(a + b)3 = a3 + 3a2b + 3ab2 + b3

   Se quisermos calcular          , podemos adotar o mesmo procedimento:

(a + b)4 = (a + b)3 (a+b) = (a3 + 3a2b + 3ab2 + b3) (a+b)

= a4 + 4a3b + 6a2b2 + 4ab3 + b4
   De modo análogo, podemos calcular as quintas e sextas potências e, de
modo geral, obter o desenvolvimento da potência              a partir da
anterior,          ou            seja,            de                     .
   Porém quando o valor de n é grande, este processo gradativo de cálculo
é                           muito                             trabalhoso.
   Existe um método para desenvolver a enésima potência de um binômio,
conhecido como binômio de Newton (Isaac Newton, matemático e físico
inglês, 1642 - 1727). Para esse método é necessário saber o que são
coeficientes binomiais, algumas de suas propriedades e o triângulo de
Pascal.



Coeficientes Binomiais

    Sendo n e p dois números naturais           , chamamos de coeficiente

binomial de classe p, do número n, o número            , que indicamos por

     (lê-se: n sobre p). Podemos escrever:




  O coeficiente binomial também é chamado de número binomial. Por
analogia com as frações, dizemos que n é o seu numerador e p, o
denominador. Podemos escrever:

Prof: Linhares e Júlio                                             Página 6
É também imediato que, para qualquer n natural, temos:




  Exemplos:




Propriedades dos coeficientes binomiais


                     Se n, p, k       e p + k = n
1ª)
                     então


  Coeficientes binomiais como esses, que tem o mesmo numerador e a
soma dos denominadores igual ao numerador, são chamados
complementares.

  Exemplos:




2ª)                      Se n, p, k   e p   p-1     0


Prof: Linhares e Júlio                                         Página 7
então


  Essa igualdade é conhecida como relação de Stifel (Michael Stifel,
matemático alemão, 1487 - 1567).

  Exemplos:




Triângulo de Pascal




      A     disposição
ordenada            dos
números binomiais,
como na tabela ao
lado, recebe o nome
de     Triângulo     de
Pascal




  Nesta tabela triangular, os números binomiais com o mesmo numerador
são escritos na mesma linha e os de mesmo denominador, na mesma
coluna.

   Por exemplo, os números binomiais                  ,   ,    e    estão na linha 3 e os

números binomiais                ,   ,   ,   , ...,   , ... estão na coluna 1.

   Substituindo cada número binomial pelo seu respectivo valor, temos:




Prof: Linhares e Júlio                                                            Página 8
Construção do triângulo de Pascal
   Para construir o triângulo do Pascal, basta lembrar as seguintes
propriedades dos números binomiais, não sendo necessário calculá-los:


1ª) Como           = 1, todos os elementos da coluna 0 são iguais a 1.


2ª) Como           = 1, o último elemento de cada linha é igual a 1.

3ª) Cada elemento do triângulo que não seja da coluna 0 nem o último de
cada         linha      é       igual        à       soma        daquele
    que está na mesma coluna e linha anterior com o elemento que se situa
à             esquerda         deste            último          (relação
    de Stifel).
      Observe os passos e aplicação da relação de Stifel para a construção
do triângulo:




Prof: Linhares e Júlio                                                   Página 9
Propriedade do triângulo de Pascal
P1 Em Qualquer linha, dois números binomiais eqüidistantes dos
extremos são iguais.




  De fato, esses binomiais são complementares.



P2 Teorema das linhas: A soma dos elementos da enésima linha é   .




  De modo geral temos:




P3 Teorema das colunas: A soma dos elementos de qualquer coluna, do
1º elemento até um qualquer, é igual ao elemento situado na coluna à
direita da considerada e na linha imediatamente abaixo.




Prof: Linhares e Júlio                                       Página 10
1 + 2 + 3 + 4 + 5 + 6 = 21

                                   1 + 4 + 10 + 20 = 35




P4 Teorema das diagonais: A soma dos elementos situados na mesma
diagonal desde o elemento da 1ª coluna até o de uma qualquer é igual ao
elemento imediatamente abaixo deste.




                                   1 + 3 + 6 + 10 + 15 = 35




Fórmula do desenvolvimento do binômio de Newton

  Como vimos, a potência da forma         , em que a,                ,é
chamada binômio de Newton. Além disso:

        quando n = 0 temos
        quando n = 1 temos
        quando n = 2 temos
        quando n = 3 temos
        quando n = 4 temos




Prof: Linhares e Júlio                                          Página 11
Observe que os coeficientes dos desenvolvimentos foram o triângulo de
Pascal. Então, podemos escrever também:




  De modo geral, quando o expoente é n, podemos escrever a fórmula do
desenvolvimento do binômio de Newton:




   Note que os expoentes de a vão diminuindo de unidade em unidade,
variando de n até 0, e os expoentes de b vão aumentando de unidade em
unidade, variando de 0 até n. O desenvolvimento de (a + b)n possui n + 1
termos.




Prof: Linhares e Júlio                                          Página 12
Fórmula do termo geral do binômio
  Observando os termos do desenvolvimento de (a + b)n, notamos

que cada um deles é da forma                                                 .


         Quando p = 0 temos o 1º termo:


         Quando p = 1 temos o 2º termo:


         Quando p = 2 temos o 3º termo:


         Quando p = 3 temos o 4º termo:


         Quando             p       = 4             temos           o       5º      termo:
   ..............................................................................
  Percebemos, então, que um termo qualquer T de ordem p + 1pode ser
expresso por:




Prof: Linhares e Júlio                                                                       Página 13
Cilindro
    Na figura abaixo, temos dois planos paralelos e distintos,     , um
círculo R contido em e uma reta r que intercepta      , mas não R:




    Para cada ponto C da região R, vamos considerar o segmento         ,
paralelo à reta r    :




     Assim, temos:




Prof: Linhares e Júlio                                         Página 14
Chamamos de cilindro, ou cilindro circular, o conjunto de todos os
segmentos  congruentes e paralelos a r.


Elementos do cilindro
     Dado o cilindro a seguir, consideramos os seguintes elementos:




        bases: os círculos de centro O e O'e raios r

        altura: a distância h entre os planos

       geratriz: qualquer segmento de extremidades nos pontos das
   circunferências das bases ( por exemplo, ) e paralelo à reta r .



Prof: Linhares e Júlio                                                Página 15
Áreas
     Num cilindro, consideramos as seguintes áreas:
a) área lateral (AL)

    Podemos observar a área lateral de um cilindro fazendo a sua
planificação:




    Assim, a área lateral do cilindro reto cuja altura é h e cujos raios dos
círculos das bases são r é um retângulo de dimensões          :




b) área da base ( AB):área do círculo de raio r




c) área total ( AT): soma da área lateral com as áreas das bases




Prof: Linhares e Júlio                                              Página 16
Volume
   Para obter o volume do cilindro, vamos usar novamente o princípio de
Cavalieri.

    Dados dois sólidos com mesma altura e um plano , se todo plano ,
paralelo ao plano , intercepta os sólidos e determina secções de mesma
área, os sólidos têm volumes iguais:




        Se 1 é um paralelepípedo retângulo, então V2 = ABh.
      Assim, o volume de todo paralelepípedo retângulo e de todo cilindro
é o produto da área da base pela medida de sua altura:

                           Vcilindro = ABh

       No caso do cilindro circular reto, a área da base é a área do círculo de
raio r         ;

portanto seu volume é:




Prof: Linhares e Júlio                                                 Página 17
Esfera
  Chamamos de esfera de centro O e raio R o conjunto de pontos do
espaço cuja distância ao centro é menor ou igual ao raio R.

    Considerando a rotação completa de um semicírculo em torno de um
eixo e, a esfera é o sólido gerado por essa rotação. Assim, ela é limitada por
uma superfície esférica e formada por todos os pontos pertencentes a essa
superfície e ao seu interior.




Volume
  O volume da esfera de raio R é dado por:




Prof: Linhares e Júlio                                                Página 18
Partes da esfera
Superfície esférica
  A superfície esférica de centro O e raio R é o conjunto de pontos do
es[aço cuja distância ao ponto O é igual ao raio R.

   Se considerarmos a rotação completa de uma semicircunferência em
torno de seu diâmetro, a superfície esférica é o resultado dessa rotação.




      A área da superfície esférica é dada por:




                          Cone circular
  Dado um círculo C, contido num plano , e um ponto V ( vértice) fora
de , chamamos de cone circular o conjunto de todos os segmentos
           .




Prof: Linhares e Júlio                                           Página 19
Elementos do cone circular
     Dado o cone a seguir, consideramos os seguintes elementos:




        altura: distância h do vértice V ao plano

      geratriz (g):segmento com uma extremidade no ponto V e outra num
   ponto da circunferência

        raio da base: raio R do círculo

       eixo de rotação:reta      determinada pelo centro do círculo e pelo
   vértice do cone



Cone reto
    Todo cone cujo eixo de rotação é perpendicular à base é chamado cone
reto, também denominado cone de revolução. Ele pode ser gerado pela
rotação completa de um triângulo retângulo em torno de um de seus
catetos.




Prof: Linhares e Júlio                                             Página 20
Da figura, e pelo Teorema de Pitágoras, temos a seguinte relação:

                                G2 = h2 +
                                R2

Secção meridiana
    A secção determinada, num cone de revolução, por um plano que
contém o eixo de rotação é chamada secção meridiana.




     Se o triângulo AVB for eqüilátero, o cone também será eqüilátero:




Prof: Linhares e Júlio                                               Página 21
Áreas
 Desenvolvendo a superfície lateral de um cone circular reto, obtemos um
setor circular de raio g e comprimento      :




        Assim, temos de considerar as seguintes áreas:

a) área lateral (AL): área do setor circular



b) área da base (AB):área do circulo do raio R



c) área total (AT):soma da área lateral com a área da base




Prof: Linhares e Júlio                                          Página 22
Volume
    Para determinar o volume do cone, vamos ver como calcular volumes
de sólidos de revolução. Observe a figura:




                                                 d = distância do
                                                 centro de gravidade
                                                 (CG)       da     sua
                                                 superfície ao eixo e

                                                 S=área da superfície




      Sabemos, pelo Teorema de Pappus - Guldin, que, quando uma
superfície gira em torno de um eixo e, gera um volume tal que:


      Vamos, então, determinar o volume do cone de revolução gerado pela
rotação de um triângulo retângulo em torno do cateto h:




    O CG do triângulo está a uma distância          do eixo de rotação.
Logo:




Prof: Linhares e Júlio                                          Página 23
CONJUNTOS NUMÉRICOS
 Conjunto dos números naturais (IN)

                 IN={0, 1, 2, 3, 4, 5,...}

  Um subconjunto importante de IN é o conjunto IN*:
  IN*={1, 2, 3, 4, 5,...}  o zero foi excluído do conjunto IN.
  Podemos considerar o conjunto dos números naturais ordenados sobre
uma reta, como mostra o gráfico abaixo:




 Conjunto dos números inteiros (Z)

           Z={..., -3, -2, -1, 0, 1, 2, 3,...}

    O conjunto IN é subconjunto de Z.
    Temos também outros subconjuntos de Z:
    Z* = Z-{0}
    Z+ = conjunto dos inteiros não negativos = {0,1,2,3,4,5,...}
    Z_ = conjunto dos inteiros não positivos = {0,-1,-2,-3,-4,-5,...}

   Observe que Z+=IN.
   Podemos considerar os números inteiros ordenados sobre uma reta,
conforme mostra o gráfico abaixo:




Prof: Linhares e Júlio                                                  Página 24
 Conjunto dos números racionais (Q)

    Os números racionais são todos aqueles que podem ser colocados na
forma de fração (com o numerador e denominador  Z). Ou seja, o
conjunto dos números racionais é a união do conjunto dos números
inteiros com as frações positivas e negativas.

             5      3 3
Então : -2,  ,  1, , 1, , por exemplo, são números racionais.
             4      5 2

Exemplos:

        3 6 9
 a)  3      
         1   2   3
       1 2 3
 b) 1   
       1 2 3


Assim, podemos escrever:

                                  a
                   Q  {x | x      , com a  Z , b  Z e b  0}
                                  b


      É interessante considerar a representação decimal de um número
racional a , que se obtém dividindo a por b.
               b
        Exemplos referentes às decimais exatas ou finitas:

             1                    5                75
                0,5                1,25            3,75
             2                    4                20

        Exemplos referentes às decimais periódicas ou infinitas:

            1                      6                           7
               0,333...              0,857142857142...          1,1666...
            3                      7                           6

     Toda decimal exata ou periódica pode ser representada na forma de
número racional.




Prof: Linhares e Júlio                                                         Página 25
 Conjunto dos números irracionais

    Os números irracionais são decimais infinitas não periódicas, ou seja,
os números que não podem ser escrito na forma de fração (divisão de dois
inteiros). Como exemplo de números irracionais, temos a raiz quadrada de
2 e a raiz quadrada de 3:
                           2  1,4142135...
                           3  1,7320508...

  Um número irracional              bastante    conhecido     é      o   número
=3,1415926535...

 Conjunto dos números reais (IR)

   Dados os conjuntos dos números racionais (Q) e dos irracionais,
definimos o conjunto dos números reais como:

       IR=Q  {irracionais} = {x|x é racional ou x é irracional}

    O diagrama abaixo mostra a relação entre os conjuntos numéricos:




      Portanto, os números naturais, inteiros, racionais e irracionais são
todos números reais. Como subconjuntos importantes de IR temos:
      IR* = IR-{0}
      IR+ = conjunto dos números reais não negativos
      IR_ = conjunto dos números reais não positivos

      Obs: entre dois números inteiros existem infinitos números reais. Por
exemplo:
 Entre os números 1 e 2 existem infinitos números reais:
   1,01 ; 1,001 ; 1,0001 ; 1,1 ; 1,2 ; 1,5 ; 1,99 ; 1,999 ; 1,9999 ...

 Entre os números 5 e 6 existem infinitos números reais:
  5,01 ; 5,02 ; 5,05 ; 5,1 ; 5,2 ; 5,5 ; 5,99 ; 5,999 ; 5,9999 ...

Prof: Linhares e Júlio                                                   Página 26
Determinantes
  Como já vimos, matriz quadrada é a que tem o mesmo número de linhas
e de colunas (ou seja, é do tipo nxn).

  A toda matriz quadrada está associado um número ao qual damos o nome
de determinante.

  Dentre as várias aplicações dos determinantes na Matemática, temos:

        resolução de alguns tipos de sistemas de equações lineares;

       cálculo da área de um triângulo situado no plano cartesiano, quando
   são conhecidas as coordenadas dos seus vértices;



Determinante de 1ª ordem
  Dada uma matriz quadrada de 1ª ordem M=[a11], o seu determinante é o
número real a11:

det M =Ia11I = a11
Observação: Representamos o determinante de uma matriz entre duas
barras verticais, que não têm o significado de módulo.

  Por exemplo:

        M= [5]           det M = 5 ou I 5 I =       M = [-3]   det M = -3 ou I -3
   5                                            I = -3




Determinante de 2ª ordem


  Dada a matriz              , de ordem 2, por definição o determinante
associado a M, determinante de 2ª ordem, é dado por:




Prof: Linhares e Júlio                                                   Página 27
Portanto, o determinante de uma matriz de ordem 2 é dado pela diferença
entre o produto dos elementos da diagonal principal e o produto dos
elementos da diagonal secundária. Veja o exemplo a seguir.




Menor complementar
  Chamamos de menor complementar relativo a um elemento aij de uma
matriz M, quadrada e de ordem n>1, o determinante MCij , de ordem n - 1,
associado à matriz obtida de M quando suprimimos a linha e a coluna que
passam por aij .
  Vejamos como determiná-lo pelos exemplos a seguir:


a) Dada a matriz             , de ordem 2, para determinar o menor
complementar relativo ao elemento a11(MC11), retiramos a linha 1 e a
coluna 1:




Da mesma forma, o menor complementar relativo ao elemento a12 é:




Prof: Linhares e Júlio                                            Página 28
b) Sendo                      , de ordem 3, temos:




Cofator
  Chamamos de cofator ou complemento algébrico relativo a um elemento
aij de uma matriz quadrada de ordem n o número Aij tal que Aij = (-1)i+j .
MCij .
  Veja:


a) Dada                  , os cofatores relativos aos elementos a11 e a12 da
matriz M são:




b) Sendo                      , vamos calcular os cofatores A22, A23 e A31:




Prof: Linhares e Júlio                                                 Página 29
Teorema de Laplace

  O determinante de uma matriz quadrada M = [aij]mxn          pode ser
obtido pela soma dos produtos dos elementos de uma fila qualquer ( linha
ou coluna) da matriz M pelos respectivos cofatores.

  Assim, fixando                    , temos:




em que   é o somatório de todos os termos de índice i, variando de 1 até
m,     .

Regra de Sarrus
  O cálculo do determinante de 3ª ordem pode ser feito por meio de um
dispositivo prático, denominado regra de Sarrus.



  Acompanhe como aplicamos essa regra para                      .



1º passo: Repetimos as duas primeiras colunas ao lado da terceira:


Prof: Linhares e Júlio                                               Página 30
2º passo: Encontramos a soma do produto dos elementos da diagonal
principal com os dois produtos obtidos pela multiplicação dos elementos
das paralelas a essa diagonal (a soma deve ser precedida do sinal positivo):




3º passo: Encontramos a soma do produto dos elementos da diagonal
secundária com os dois produtos obtidos pela multiplicação dos elementos
das paralelas a essa diagonal ( a soma deve ser precedida do sinal
negativo):




Assim:




Prof: Linhares e Júlio                                               Página 31
Observação: Se desenvolvermos esse determinante de 3ª ordem aplicando o
Teorema de Laplace, encontraremos o mesmo número real.



Determinante de ordem n > 3
  Vimos que a regra de Sarrus é válida para o cálculo do determinante de
uma matriz de ordem 3. Quando a matriz é de ordem superior a 3, devemos
empregar o Teorema de Laplace para chegar a determinantes de ordem 3 e
depois aplicar a regra de Sarrus.

Propriedades dos determinantes
  Os demais associados a matrizes quadradas de ordem n apresentam as
seguintes propriedades:

P1 ) Quando todos os elementos de uma fila ( linha ou coluna) são nulos, o
determinante dessa matriz é nulo.

Exemplo:




P2) Se duas filas de uma matriz são iguais, então seu determinante é nulo.

Exemplo:




P3) Se duas filas paralelas de uma matriz são proporcionais, então seu
determinante é nulo.
Exemplo:

Prof: Linhares e Júlio                                               Página 32
P4) Se os elementos de uma fila de uma matriz são combinações lineares
dos elementos correspondentes de filas paralelas, então seu determinante é
nulo.

Exemplos:




P5 ) Teorema de Jacobi: o determinante de uma matriz não se altera
quando somamos aos elementos de uma fila uma combinação linear dos
elementos correspondentes de filas paralelas.

Exemplo:




Substituindo a 1ª coluna pela soma dessa mesma coluna com o dobro da 2ª,
temos:




P6) O determinante de uma matriz e o de sua transposta são iguais.

Exemplo:




Prof: Linhares e Júlio                                               Página 33
P7) Multiplicando por um número real todos os elementos de uma fila em
uma matriz, o determinante dessa matriz fica multiplicado por esse número.

Exemplos:




P8) Quando trocamos as posições de duas filas paralelas, o determinante de
uma matriz muda de sinal.
Exemplo:




P9) Quando, em uma matriz, os elementos acima ou abaixo da diagonal
principal são todos nulos, o determinante é igual ao produto dos elementos
dessa diagonal.

Exemplos:


Prof: Linhares e Júlio                                            Página 34
P10) Quando, em uma matriz, os elementos acima ou abaixo da diagonal
secundária são todos nulos, o determinante é igual ao produto dos

elementos dessa diagonal multiplicado por                 .

Exemplos:




P11)     Para     A      e   B    matrizes   quadradas   de   mesma   ordem   n,




                                 . Como:

Exemplo:




P12)

Exemplo:




Prof: Linhares e Júlio                                                  Página 35
Equações algébricas
                             (com uma variável)

   Introdução
   Equação é toda sentença matemática aberta que exprime uma relação de
igualdade. A palavra equação tem o prefixo equa, que em latim quer dizer
"igual". Exemplos:

2x + 8 = 0

5x - 4 = 6x + 8
3a - b - c = 0

Não são equações:

4 + 8 = 7 + 5 (Não é uma sentença aberta)
x - 5 < 3 (Não é igualdade)

          (não é sentença aberta, nem igualdade)
A equação geral do primeiro grau:
ax+b = 0
onde a e b são números conhecidos e a > 0, se resolve de maneira simples:
subtraindo b dos dois lados, obtemos:

ax = -b

dividindo agora por a (dos dois lados), temos:




  Considera a equação 2x - 8 = 3x -10


Prof: Linhares e Júlio                                           Página 36
A letra é a incógnita da equação. A palavra incógnita significa "
desconhecida".

  Na equação acima a incógnita é x; tudo que antecede o sinal da igualdade
denomina-se 1º membro, e o que sucede, 2º membro.




  Qualquer parcela, do 1º ou do 2º membro, é um termo da equação.




       Equação do 1º grau na incógnita x é toda equação que pode ser
       escrita na forma ax=b, sendo a e b números racionais, com a
       diferente de zero.

Conjunto Verdade e Conjunto Universo de uma Equação
   Considere o conjunto A = {0, 1, 2, 3, 4, 5} e a equação x + 2 = 5.

   Observe que o número 3 do conjunto A é denominado conjunto
universo da equação e o conjunto {3} é o conjunto verdade dessa mesma
equação.



   Observe este outro exemplo:

            Determine os números inteiros que satisfazem a equação x² = 25

            O conjunto dos números inteiro é o conjunto universo da equação.

Prof: Linhares e Júlio                                                  Página 37
Os números -5 e 5, que satisfazem a equação, formam o conjunto
verdade, podendo ser indicado por: V = {-5, 5}.
   Daí concluímos que:

          Conjunto Universo é o conjunto de todos os valores que
          variável pode assumir. Indica-se por U.



          Conjunto verdade é o conjunto dos valores de U, que
          tornam verdadeira a equação . Indica-se por V.



Observações:

        O conjunto verdade é subconjunto do conjunto universo.




      Não sendo citado o conjunto universo, devemos considerar como
   conjunto universo o conjunto dos números racionais.



      O conjunto verdade é também conhecido por conjunto solução e
   pode ser indicado por S.

                         Raízes de uma equação

  Os elementos do conjunto verdade de uma equação são chamados raízes
da equação.

  Para verificar se um número é raiz de uma equação, devemos obedecer à
seguinte seqüência:

        Substituir a incógnita por esse número.

        Determinar o valor de cada membro da equação.
      Verificar a igualdade, sendo uma sentença verdadeira, o número
   considerado é raiz da equação.


Prof: Linhares e Júlio                                             Página 38
Exemplos:

          Verifique quais dos elementos do conjunto universo são raízes
das equações abaixo, determinando em cada caso o conjunto verdade.



        Resolva a equação x - 2 = 0, sendo U = {0, 1, 2, 3}.

                          Para x = 0 na equação x - 2 = 0 temos: 0 - 2 = 0
=> -2 = 0. (F)
                          Para x = 1 na equação x - 2 = 0 temos: 1 - 2 = 0
=> -1 = 0. (F)

                          Para x = 2 na equação x - 2 = 0 temos: 2 - 2 = 0
=> 0 = 0. (V)

                          Para x = 3 na equação x - 2 = 0 temos: 3 - 2 = 0
=> 1 = 0. (F)

   Verificamos que 2 é raiz da equação x - 2 = 0, logo V = {2}.



        Resolva a equação 2x - 5 = 1, sendo U = {-1, 0, 1, 2}.



                          Para x = -1 na equação 2x - 5 = 1 temos: 2 . (-1) -
5 = 1 => -7 = 1. (F)

                          Para x = 0 na equação 2x - 5 = 1 temos: 2 . 0 - 5 =
1 => -5 = 1. (F)

                          Para x = 1 na equação 2x - 5 = 1 temos: 2 . 1 - 5 =
1 => -3 = 1. (F)

                          Para x = 2 na equação 2x - 5 = 1 temos: 2 . 2 - 5 =
1 => -1 = 1. (F)



   A equação 2x - 5 = 1 não possui raiz em U, logo V = Ø.




Prof: Linhares e Júlio                                               Página 39
Função de 1º grau - Afim
    Definição

 Chama-se função polinomial do 1º grau, ou função afim, a qualquer
função f de IR em IR dada por uma lei da forma f(x) = ax + b, onde a e b
são números reais dados e a 0.

Na função f(x) = ax + b, o número a é chamado de coeficiente de x e o
número b é chamado termo constante.

Veja alguns exemplos de funções polinomiais do 1º grau:
f(x)    =    5x     -   3,    onde       a     =    5    e    b    =       -   3
f(x)    =    -2x     - 7,      onde      a     =    -2   e    b    =       -   7
f(x) = 11x, onde a = 11 e b = 0



Gráfico
  O gráfico de uma função polinomial do 1º grau, y = ax + b, com a 0, é
uma reta oblíqua aos eixos Ox e Oy.
     Exemplo:

    Vamos construir o gráfico da função y = 3x - 1:
  Como o gráfico é uma reta, basta obter dois de seus pontos e ligá-los
com o auxílio de uma régua:

     a)   Para x = 0, temos y = 3 · 0 - 1 = -1; portanto, um ponto é (0, -1).

     b)   Para y = 0, temos 0 = 3x - 1; portanto,        e outro ponto é
.


   Marcamos os pontos (0, -1) e              no plano cartesiano e ligamos os
dois com uma reta.




Prof: Linhares e Júlio                                                 Página 40
x               y
           0               -1
                           0




   Já vimos que o gráfico da função afim y = ax + b é uma reta.
  O coeficiente de x, a, é chamado coeficiente angular da reta e, como
veremos adiante, a está ligado à inclinação da reta em relação ao eixo Ox.
  O termo constante, b, é chamado coeficiente linear da reta. Para x = 0,
temos y = a · 0 + b = b. Assim, o coeficiente linear é a ordenada do ponto
em que a reta corta o eixo Oy.

Zero e Equação do 1º Grau
  Chama-se zero ou raiz da função polinomial do 1º grau f(x) = ax + b, a
0, o número real x tal que f(x) = 0.

  Temos:


  f(x) = 0               ax + b = 0

  Vejamos alguns exemplos:
    1. Obtenção            do    zero       da    função   f(x)   =   2x    -    5:

                                       f(x) = 0      2x - 5 = 0

    2. Cálculo            da    raiz     da    função  g(x)   =  3x        +   6:
                                       g(x) = 0       3x + 6 = 0           x = -2


    3. Cálculo da abscissa do ponto em que o gráfico de h(x) = -2x + 10
       corta          o           eixo            das           abicissas:
       O ponto em que o gráfico corta o eixo dos x é aquele em que h(x) =


Prof: Linhares e Júlio                                                     Página 41
0;                                                            então:
             h(x) = 0         -2x + 10 = 0        x=5



Crescimento e decrescimento
  Consideremos a função do 1º grau y = 3x - 1. Vamos atribuir valores cada
vez maiores a x e observar o que ocorre com y:




 x            -3         -2      -1          0     1    2        3
 y            -10        -7      -4          -1    2    5        8



       Notemos que, quando aumentos o valor de x, os correspondentes
       valores de y também aumentam. Dizemos, então que a
          função    y      =       3x  -      1     é      crescente.
     Observamos novamente seu gráfico:




Regra geral:

a função do 1º grau f(x) = ax + b é crescente quando o coeficiente de x é
positivo                     (a                   >                     0);
a função do 1º grau f(x) = ax + b é decrescente quando o coeficiente de x é
negativo (a < 0);



Prof: Linhares e Júlio                                               Página 42
Justificativa:

      para a > 0: se x1 < x2, então ax1 < ax2. Daí, ax1 + b < ax2 + b, de onde
   vem f(x1) < f(x2).
      para a < 0: se x1 < x2, então ax1 > ax2. Daí, ax1 + b > ax2 + b, de onde
   vem f(x1) > f(x2).

Sinal
  Estudar o sinal de uma qualquer y = f(x) é determinar os valor de x para
os quais y é positivo, os valores de x para os quais y é zero e os valores de
x         para         os         quais         y         é         negativo.
   Consideremos uma função afim y = f(x) = ax + b vamos estudar seu

sinal. Já vimos que essa função se anula pra raiz            . Há dois casos
possíveis:
 1º) a > 0 (a função é crescente)


        y>0              ax + b > 0   x>


        y>0              ax + b < 0   x<
   Conclusão: y é positivo para valores de x maiores que a raiz; y é
negativo para valores de x menores que a raiz




2º) a < 0 (a função é decrescente)


        y>0          ax + b > 0       x<

Prof: Linhares e Júlio                                                Página 43
y>0          ax + b < 0    x<


Conclusão: y é positivo para valores de x menores que a raiz; y é negativo
para valores de x maiores que a raiz.




Prof: Linhares e Júlio                                            Página 44
EQUAÇÕES EXPONENCIAIS
      Chamamos de equações exponenciais toda equação na qual a
incógnita aparece em expoente.

Exemplos de equações exponenciais:
1) 3x =81 (a solução é x=4)
2) 2x-5=16 (a solução é x=9)
3) 16x-42x-1-10=22x-1 (a solução é x=1)
4) 32x-1-3x-3x-1+1=0 (as soluções são x’=0 e x’’=1)

   Para resolver equações exponenciais, devemos realizar dois passos
importantes:
   1º) redução dos dois membros da equação a potências de mesma
base;
   2º) aplicação da propriedade:

                         a m  a n  m  n (a  1 e a  0)


EXERCÍCIOS RESOLVIDOS:

1) 3x=81
Resolução: Como 81=34, podemos escrever 3x = 34
E daí, x=4.

2) 9x = 1
Resolução: 9x = 1  9x = 90 ; logo x=0.

         x
   3    81
3)   
   4   256
                         x              x           x     4
            3    81               3  34   3   3
Resolução :                        4       ; então x  4.
            4   256               4  4    4   4


4) 3 x  4 27
                                                3
                                                          3
Resolução : 3  27  3  3  3  3 ; logo x 
                  x      4      x   4   3   x   4
                                                          4

5) 23x-1 = 322x
Prof: Linhares e Júlio                                             Página 45
Resolução: 23x-1 = 322x  23x-1 = (25)2x  23x-1 = 210x ; daí 3x-1=10,
de onde x=-1/7.

6) Resolva a equação 32x–6.3x–27=0.
Resolução: vamos resolver esta equação através de uma transformação:
32x–6.3x–27=0  (3x)2-6.3x–27=0
Fazendo 3x=y, obtemos:
y2-6y–27=0 ; aplicando Bhaskara encontramos  y’=-3 e y’’=9
Para achar o x, devemos voltar os valores para a equação auxiliar 3x=y:

y’=-3  3x’ = -3  não existe x’, pois potência de base positiva é
positiva
y’’=9  3x’’ = 9  3x’’ = 32  x’’=2

Portanto a solução é x=2


                           FUNÇÃO EXPONENCIAL

       Chamamos de funções exponenciais aquelas nas quais temos a
variável aparecendo em expoente.
       A função f:IRIR+ definida por f(x)=ax, com a  IR+ e a1, é
chamada função exponencial de base a. O domínio dessa função é o
conjunto IR (reais) e o contradomínio é IR+ (reais positivos, maiores que
zero).


              GRÁFICO CARTESIANO DA FUNÇÃO EXPONENCIAL

        Temos 2 casos a considerar:
         quando a>1;
         quando 0<a<1.

        Acompanhe os exemplos seguintes:

1) y=2x (nesse caso, a=2, logo a>1)
   Atribuindo alguns valores a x e calculando os correspondentes valores
   de y, obtemos a tabela e o gráfico abaixo:




Prof: Linhares e Júlio                                             Página 46
X       -2    -1       0         1      2
                 y       1/4   1/2      1         2      4




2) y=(1/2)x (nesse caso, a=1/2, logo 0<a<1)
   Atribuindo alguns valores a x e calculando os correspondentes valores
   de y, obtemos a tabela e o gráfico abaixo:

                 X       -2    -1       0          1     2
                 Y       4      2       1         1/2   1/4




    Nos dois exemplos, podemos observar que
    a) o gráfico nunca intercepta o eixo horizontal; a função não tem
       raízes;
    b) o gráfico corta o eixo vertical no ponto (0,1);
    c) os valores de y são sempre positivos (potência de base positiva é
       positiva), portanto o conjunto imagem é Im=IR+.

    Além disso, podemos estabelecer o seguinte:




Prof: Linhares e Júlio                                          Página 47
a>1                                       0<a<1




       f(x) é crescente e Im=IR+                            f(x) é decrescente e Im=IR+
  Para quaisquer x1 e x2 do domínio:                     Para quaisquer x1 e x2 do domínio:
 x2>x1  y2>y1 (as desigualdades têm                    x2>x1  y2<y1 (as desigualdades têm
            mesmo sentido)                                       sentidos diferentes)


                                      INEQUAÇÕES EXPONENCIAIS

      Chamamos de inequações exponenciais toda inequação na qual a
incógnita aparece em expoente.

Exemplos de inequações exponenciais:

1) 3 x  81 (a solução é x  4)
                      1
2) 2 2x-2  2 x
                  2
                                (que é satisfeita para todo x real)
           x               3
   4      4
3)              (que é satisfeita para x  -3)
   5      5
4) 25 x - 150.5 x  3125  0 (que é satisfeita para 2  x  3)


   Para resolver inequações exponenciais, devemos realizar dois passos
importantes:
   1º) redução dos dois membros da inequação a potências de mesma
base;
   2º) aplicação da propriedade:

                                a>1                                    0<a<1
               am > an  m>n                                      am > an  m<n
(as desigualdades têm mesmo sentido)                        (as desigualdades têm sentidos
                                                                      diferentes)
Prof: Linhares e Júlio                                                             Página 48
EXERCÍCIO RESOLVIDO:

                              11
1) 4 x 1  4 x  4 x 1 
                              4
Resolução :
                              4x                   11
A inequação pode ser escrita      4 x  4 x .4       .
                               4                   4
M ultiplicando ambos os lados por 4 temos :
4 x  4.4 x  16.4 x  11 , ou seja :
(1  4  16).4 x  11  -11.4 x  11 e daí, 4 x  1
Porém, 4 x  1  4 x  4 0.
Como a base (4) é maior que 1, obtemos :
4 x  40  x  0
Portanto S  IR - (reais negativos)




Prof: Linhares e Júlio                                     Página 49
FUNÇÃO LOGARÍTMICA
       A função f:IR+IR definida por f(x)=logax, com a1 e a>0, é
chamada função logarítmica de base a. O domínio dessa função é o
conjunto IR+ (reais positivos, maiores que zero) e o contradomínio é IR
(reais).


              GRÁFICO CARTESIANO DA FUNÇÃO LOGARÍTMICA

        Temos 2 casos a considerar:
         quando a>1;
         quando 0<a<1.

      Acompanhe nos exemplos seguintes, a construção do gráfico em
cada caso:

3) y=log2x (nesse caso, a=2, logo a>1)
   Atribuindo alguns valores a x e calculando os correspondentes valores
   de y, obtemos a tabela e o gráfico abaixo:


                  x      1/4   1/2     1        2        4
                  y      -2    -1      0        1        2




4) y=log(1/2)x (nesse caso, a=1/2, logo 0<a<1)
   Atribuindo alguns valores a x e calculando os correspondentes valores
   de y, obtemos a tabela e o gráfico abaixo:




Prof: Linhares e Júlio                                          Página 50
x        1/4   1/2     1        2         4
                  y         2     1      0        -1       -2




    Nos dois exemplos, podemos observar que
    d) o gráfico nunca intercepta o eixo vertical;
    e) o gráfico corta o eixo horizontal no ponto (1,0). A raiz da função é
       x=1;
    f) y assume todos os valores reais, portanto o conjunto imagem é
       Im=IR.

    Além disso, podemos estabelecer o seguinte:

                         a>1                           0<a<1




       f(x) é crescente e Im=IR              f(x) é decrescente e Im=IR
  Para quaisquer x1 e x2 do domínio:     Para quaisquer x1 e x2 do domínio:
 x2>x1  y2>y1 (as desigualdades têm    x2>x1  y2<y1 (as desigualdades têm
            mesmo sentido)                       sentidos diferentes)




Prof: Linhares e Júlio                                             Página 51
EQUAÇÕES LOGARÍTMICAS

       Chamamos de equações logarítmicas toda equação que envolve
logaritmos com a incógnita aparecendo no logaritmando, na base ou em
ambos.

Exemplos de equações logarítmicas:
7) log3x =5 (a solução é x=243)
8) log(x2-1) = log 3 (as soluções são x’=-2 e x’’=2)
9) log2(x+3) + log2(x-3) = log27 (a solução é x=4)
10) logx+1(x2-x)=2 (a solução é x=-1/3)

Alguns exemplos resolvidos:
1) log3(x+5) = 2
   Resolução: condição de existência: x+5>0 => x>-5
   log3(x+5) = 2 => x+5 = 32 => x=9-5 => x=4
   Como x=4 satisfaz a condição de existência, então o conjunto
solução é S={4}.

2) log2(log4 x) = 1
   Resolução: condição de existência: x>0 e log4x>0
   log2(log4 x) = 1 ; sabemos que 1 = log2(2), então
   log2(log4x) = log2(2) => log4x = 2 => 42 = x => x=16
   Como x=16 satisfaz as condições de existência, então o
conjunto solução é S={16}.

3) Resolva o sistema:
log x  log y  7

3. log x  2. log y  1

Resolução: condições de existência: x>0 e y>0
Da primeira equação temos:
log x+log y=7 => log y = 7-log x
Substituindo log y na segunda equação temos:
3.log x – 2.(7-log x)=1 => 3.log x-14+2.log x = 1 => 5.log x = 15 =>

=> log x =3 => x=103
Substituindo x= 103 em log y = 7-log x temos:

Prof: Linhares e Júlio                                          Página 52
log y = 7- log 103 => log y = 7-3 => log y =4 => y=104.
Como essas raízes satisfazem as condições de existência, então o conjunto
solução é S={(103;104)}.


                               INEQUAÇÕES LOGARÍTMICAS

       Chamamos de inequações logarítmicas toda inequação que envolve
logaritmos com a incógnita aparecendo no logaritmando, na base ou em
ambos.

Exemplos de inequações logarítmicas:
  1) log2x > 0 (a solução é x>1)
  2) log4(x+3)  1 (a solução é –3<x1)

   Para resolver inequações logarítmicas, devemos realizar dois passos
importantes:
   1º) redução dos dois membros da inequação a logaritmos de mesma
base;
   2º) aplicação da propriedade:

                         a>1                             0<a<1
    logam > logan  m>n>0                     logam > logan  0<m<n
(as desigualdades têm mesmo sentido)          (as desigualdades têm sentidos
                                                        diferentes)


EXERCÍCIOS RESOLVIDOS:

1) log2(x+2) > log28
   Resolução:
   Condições de existência: x+2>0, ou seja, x>-2 (S1)
   Como a base (2) é maior que 1, temos:
   x+2>8 e, daí, x>6 (S2)
   O conjunto solução é S= S1  S2 = {x  IR| x>6}.
   Portanto a solução final é a intersecção de S1 e S2, como está
representado logo abaixo no desenho:




Prof: Linhares e Júlio                                               Página 53
2) log2(log3x)  0
   Resolução:
   Condições de existência: x>0 e log3x>0
   Como log21=0, a inequação pode ser escrita assim:
   log2(log3x)  log21
   Sendo a base (2) maior que 1, temos: log3x  1.
   Como log33 = 1, então, log3x  log33 e, daí, x  3, porque a base (3) é
   maior que 1.
   As condições de existência estão satisfeitas, portanto S={x  IR| x  3}.




Prof: Linhares e Júlio                                              Página 54
Função Quadrática
 Definição

  Chama-se função quadrática, ou função polinomial do 2º grau, qualquer
função f de IR em IR dada por uma lei da forma f(x) = ax2 + bx + c, onde a,
b      e       c     são       números       reais      e      a         0.
  Vejamos alguns exemplos de função quadráticas:

    1.    f(x) = 3x2 - 4x + 1, onde a = 3, b = - 4 e c = 1
    2.    f(x) = x2 -1, onde a = 1, b = 0 e c = -1
    3.    f(x) = 2x2 + 3x + 5, onde a = 2, b = 3 e c = 5
    4.    f(x) = - x2 + 8x, onde a = 1, b = 8 e c = 0
    5.    f(x) = -4x2, onde a = - 4, b = 0 e c = 0



Gráfico
   O gráfico de uma função polinomial do 2º grau, y = ax2 + bx + c, com a
  0, é uma curva chamada parábola.
Exemplo:
     Vamos construir o gráfico da função y = x 2 + x:
   Primeiro atribuímos a x alguns valores, depois calculamos o valor
correspondente de y e, em seguida, ligamos os pontos assim obtidos.



         x               y
         -3              6
         -2              2
         -1              0


         0               0
         1               2
         2               6




Prof: Linhares e Júlio                                             Página 55
Observação:

  Ao construir o gráfico de uma função quadrática y = ax2 + bx + c,
notaremos sempre que:

        se a > 0, a parábola tem a concavidade voltada para cima;

        se a < 0, a parábola tem a concavidade voltada para baixo;



Zero e Equação do 2º Grau
  Chama-se zeros ou raízes da função polinomial do 2º grau f(x) = ax2 +
bx + c , a 0, os números reais x tais que f(x) = 0.

  Então as raízes da função f(x) = ax2 + bx + c são as soluções da equação
do 2º grau ax2 + bx + c = 0, as quais são dadas pela chamada fórmula de
Bhaskara:




   Temos:




Observação

  A quantidade de raízes reais de uma função quadrática depende do valor
obtido para o radicando             , chamado discriminante, a saber:

        quando           é positivo, há duas raízes reais e distintas;

        quando           é zero, há só uma raiz real;
        quando           é negativo, não há raiz real.

Coordenadas do vértice da parábola
  Quando a > 0, a parábola tem concavidade voltada para cima e um ponto
de mínimo V; quando a < 0, a parábola tem concavidade voltada para baixo
e um ponto de máximo V.


Prof: Linhares e Júlio                                                   Página 56
Em qualquer caso, as coordenadas de V são   . Veja os gráficos:




Prof: Linhares e Júlio                                  Página 57
Imagem
   O conjunto-imagem Im da função y = ax2 + bx + c, a     0, é o conjunto
dos valores que y pode assumir. Há duas possibilidades:

1ª - quando a > 0,




                             a>0




2ª quando a < 0,




                             a<0




Prof: Linhares e Júlio                                           Página 58
Construção da Parábola
  É possível construir o gráfico de uma função do 2º grau sem montar a
tabela de pares (x, y), mas seguindo apenas o roteiro de observação
seguinte:

    1. O valor do coeficiente a define a concavidade da parábola;

    2. Os zeros definem os pontos em que a parábola intercepta o eixo dos
       x;


    3. O vértice V               indica o ponto de mínimo (se a > 0), ou
       máximo (se a< 0);
    4. A reta que passa por V e é paralela ao eixo dos y é o eixo de
       simetria da parábola;
    5. Para x = 0 , temos y = a · 02 + b · 0 + c = c; então (0, c) é o ponto em
       que a parábola corta o eixo dos y.

Sinal
  Consideramos uma função quadrática y = f(x) = ax2 + bx + c e
determinemos os valores de x para os quais y é negativo e os valores de x
para          os         quais          y           é           positivos.
                                            2
   Conforme o sinal do discriminante = b - 4ac, podemos ocorrer os
seguintes casos:
1º- >0
  Nesse caso a função quadrática admite dois zeros reais distintos (x1

Prof: Linhares e Júlio                                                 Página 59
x2). a parábola intercepta o eixo Ox em dois pontos e o sinal da função é o
indicado nos gráficos abaixo:




                                 quando a > 0

y     >     0                (x       <         x1       ou       x       >        x2)
y < 0 x1 < x < x2




                                 quando a < 0

y            >          0            x1              <        x       <             x2
y<0         (x < x1 ou x > x2)


Prof: Linhares e Júlio                                                        Página 60
2º -      =0




                         quando a > 0




                         quando a < 0




Prof: Linhares e Júlio                  Página 61
3º -      <0




                         quando a > 0




Prof: Linhares e Júlio                  Página 62
quando a < 0




                GEOMETRIA ANALÍTICA
Retas
Introdução
   Entre os pontos de uma reta e os números reais existe uma
correspondência biunívoca, isto é, a cada ponto de reta corresponde um
único número real e vice-versa.
  Considerando uma reta horizontal x, orientada da esquerda para direita
(eixo), e determinando um ponto O dessa reta ( origem) e um segmento u,
unitário e não-nulo, temos que dois números inteiros e consecutivos
determinam sempre nesse eixo um segmento de reta de comprimento u:




Medida algébrica de um segmento
  Fazendo corresponder a dois pontos, A e B, do eixo x os números reais xA
e xB , temos:




Prof: Linhares e Júlio                                            Página 63
A medida algébrica de um segmento orientado é o número real que
corresponde à diferença entre as abscissas da extremidade e da origem
desse segmento.



Plano cartesiano
  A geometria analítica teve como principal idealizador o filósofo francês
René Descartes ( 1596-1650). Com o auxílio de um sistema de eixos
associados a um plano, ele faz corresponder a cada ponto do plano um par
ordenado e vice-versa.

   Quando os eixos desse sistemas são perpendiculares na origem, essa
correspondência determina um sistema cartesiano ortogonal ( ou plano
cartesiano). Assim, há uma reciprocidade entre o estudo da geometria (
ponto, reta, circunferência) e da Álgebra ( relações, equações etc.),
podendo-se representar graficamente relações algébricas e expressar
algebricamente representações gráficas.
  Observe o plano cartesiano nos quadros quadrantes:




Prof: Linhares e Júlio                                            Página 64
Exemplos:

        A(2, 4) pertence ao 1º quadrante (xA > 0 e yA > 0)
        B(-3, -5) pertence ao 3º quadrante ( xB < 0 e yB < 0)

Observação: Por convenção, os pontos localizados sobre os eixos não estão
em nenhum quadrante.



Distância entre dois pontos
  Dados os pontos A(xA, yA) e B(xB, yB) e sendo dAB a distância entre eles,
temos:




  Aplicando o teorema de Pitágoras ao triângulo retângulo ABC, vem:




Como exemplo, vamos determinar a distância entre os pontos A(1, -1) e
B(4, -5):




Prof: Linhares e Júlio                                             Página 65
Equações de uma reta
Equação geral
  Podemos estabelecer a equação geral de uma reta a partir da condição de
alinhamento de três pontos.
   Dada uma reta r, sendo A(xA, yA) e B(xB, yB) pontos conhecidos e
distintos de r e P(x,y) um ponto genérico, também de r, estando A, B e P
alinhados, podemos escrever:




   Fazendo yA - yB = a, xB - xA = b e xAyB - xByA=c, como a e b não são
simultaneamente nulos         , temos:


                            ax + by + c = 0


(equação geral da reta r)
  Essa equação relaciona x e y para qualquer ponto P genérico da reta.
Assim, dado o ponto P(m, n):

Prof: Linhares e Júlio                                           Página 66
se am + bn + c = 0, P é o ponto da reta;

        se am + bn + c       0, P não é ponto da reta.
                               Acompanhe os exemplos:

      Vamos considerar a equação geral da reta r que passa por A(1, 3) e
   B(2, 4).

      Considerando um ponto P(x, y) da reta, temos:




      Vamos verificar se os pontos P(-3, -1) e Q(1, 2) pertencem à reta r
   do exemplo anterior. Substituindo as coordenadas de P em x - y + 2 = 0,
   temos:

-3 - (-1) + 2 = 0        -3 + 1 + 2 = 0

  Como a igualdade é verdadeira, então P r.
  Substituindo as coordenadas de Q em x - y + 2 = 0, obtemos:

1-2+2          0
  Como a igualdade não é verdadeira, então Q r.




Prof: Linhares e Júlio                                            Página 67
Geometria Analítica: Circunferência
Equações da circunferência
Equação reduzida
    Circunferência é o conjunto de todos os pontos de um plano
eqüidistantes de um ponto fixo, desse mesmo plano, denominado centro da
circunferência:




   Assim, sendo C(a, b) o centro e P(x, y) um ponto qualquer da
circunferência, a distância de C a P(dCP) é o raio dessa circunferência.
Então:




   Portanto, (x - a)2 + (y - b)2 =r2 é a equação reduzida da circunferência e
permite determinar os elementos essenciais para a construção da
circunferência: as coordenadas do centro e o raio.


Prof: Linhares e Júlio                                               Página 68
Observação: Quando o centro da circunfer6encia estiver na origem (
C(0,0)), a equação da circunferência será x2 + y2 = r2 .



Equação geral
   Desenvolvendo a equação reduzida, obtemos a equação geral da
circunferência:




  Como exemplo, vamos determinar a equação geral da circunferência de
centro C(2, -3) e raio r = 4.

  A equação reduzida da circunferência é:
( x - 2 )2 +( y + 3 )2 = 16

  Desenvolvendo os quadrados dos binômios, temos:




              Geometria Analítica - Cônicas
Elipse
  Considerando, num plano , dois pontos distintos, F1 e F2 , e sendo 2a
um número real maior que a distância entre F1 e F2, chamamos de elipse o
conjunto dos pontos do plano tais que a soma das distâncias desses
pontos a F1 e F2 seja sempre igual a 2a.

  Por exemplo, sendo P, Q, R, S, F1 e F2 pontos de um mesmo plano e F1F2
< 2a, temos:




Prof: Linhares e Júlio                                          Página 69
A figura obtida é uma elipse.
Observações:

1ª) A Terra descreve uma trajetória elíptica em torno do sol, que é um dos
focos dessa trajetória.
    A lua em torno da terra e os demais satélites em relação a seus
respectivos planetas também apresentam esse comportamento.

2ª) O cometa de Halley segue uma órbita elíptica, tendo o Sol como um dos
focos.

3ª) As elipses são chamadas cônicas porque ficam configuradas pelo corte
feito em um cone circular reto por um plano oblíquo em relação à sua base.




Elementos
   Observe a elipse a seguir. Nela, consideramos os seguintes elementos:




Prof: Linhares e Júlio                                             Página 70
focos : os pontos F1 e F2

        centro: o ponto O, que é o ponto médio de

        semi-eixo maior: a
        semi-eixo menor: b

        semidistância focal: c

        vértices: os pontos A1, A2, B1, B2

        eixo maior:

        eixo menor:

        distância focal:

Relação fundamental
   Na figura acima, aplicando o Teorema de Pitágoras ao tri6angulo OF2B2
, retângulo em O, podemos escrever a seguinte relação fundamental:


                                    a2 =b2 + c2


Excentricidade
   Chamamos de excentricidade o número real e tal que:




  Pela definição de elipse, 2c < 2a, então c < a e, conseqüentemente, 0 < e
< 1.

Observação:Quando os focos são muito próximos, ou seja, c é muito
pequeno, a elipse se aproxima de uma circunferência.




Prof: Linhares e Júlio                                             Página 71
Equações
  Vamos considerar os seguintes casos:
a) elipse com centro na origem e eixo maior horizontal

  Sendo c a semidistância focal, os focos da elipse são F1(-c, 0) e F2(c, 0):




   Aplicando a definição de elipse                   , obtemos a equação da
elipse:




b) elipse com centro na origem e eixo maior vertical
  Nessas condições, a equação da elipse é:




Prof: Linhares e Júlio                                                 Página 72
Hipérbole
  Considerando, num plano , dois pontos distintos, F1 e F2 , e sendo 2a
um número real menor que a distância entre F1 e F2 , chamamos de
hipérbole o conjunto dos pontos do plano tais que o módulo da diferença
das dist6ancias desses pontos a F1 e F2 seja sempre igual a 2a.
  Por exemplo, sendo P, Q, R, S, F1 e F2 pontos de um mesmo plano e
F1F2 = 2c, temos:




Prof: Linhares e Júlio                                         Página 73
A figura obtida é uma hipérbole.

Observação:Os dois ramos da
hipérbole são determinados por um
plano paralelo ao eixo de simetria de
dois cones circulares retos e opostos
pelo vértice:




Parábola

  Dados uma reta d e um ponto F           , de um plano , chamamos de
parábola o conjunto de pontos do plano   eqüidistantes de F e d.

  Assim, sendo, por exemplo, F, P, Q e R pontos de um plano e d uma
reta desse mesmo plano, de modo que nenhum ponto pertença a d, temos:




Observações:

Prof: Linhares e Júlio                                        Página 74
1ª) A parábola é obtida seccionando-se obliquamente um cone circular reto:




2ª) Os telescópios refletores mais simples têm espelhos com secções planas
parabólicas.

3ª) As trajetórias de alguns cometas são parábolas, sendo que o Sol ocupa o
foco.
4ª) A superfície de um líquido contido em um cilindro que gira em torno de
seu eixo com velocidade constante é parabólica.




Prof: Linhares e Júlio                                             Página 75
Matrizes
Introdução
  O crescente uso dos computadores tem feito com que a teoria das
matrizes seja cada vez mais aplicada em áreas como Economia,
Engenharia, Matemática, Física, dentre outras. Vejamos um exemplo.

  A tabela a seguir representa as notas de três alunos em uma etapa:


                         Química Inglês    Literatura Espanhol


               A         8       7         9          8


               B         6       6         7          6


               C         4       8         5          9


  Se quisermos saber a nota do aluno B em Literatura, basta procurar o
número que fica na segunda linha e na terceira coluna da tabela.
  Vamos agora considerar uma tabela de números dispostos em linhas e
colunas, como no exemplo acima, mas colocados entre parênteses ou
colchetes:




  Em tabelas assim dispostas, os números são os elementos. As linhas são
enumeradas de cima para baixo e as colunas, da esquerda para direita:




Prof: Linhares e Júlio                                                 Página 76
Tabelas com m linhas e n colunas ( m e n números naturais diferentes de
0) são denominadas matrizes m x n. Na tabela anterior temos, portanto,
uma matriz 3 x 3.

  Veja mais alguns exemplos:


                         é uma matriz do tipo 2 x 3



                     é uma matriz do tipo 2 x 2



Notação geral
   Costuma-se representar as matrizes por letras maiúsculas e seus
elementos por letras minúsculas, acompanhadas por dois índices que
indicam, respectivamente, a linha e a coluna que o elemento ocupa.
  Assim, uma matriz A do tipo m x n é representada por:




ou, abreviadamente, A = [aij]m x n, em que i e j representam,
respectivamente, a linha e a coluna que o elemento ocupa. Por exemplo, na
matriz anterior, a23 é o elemento da 2ª linha e da 3ª coluna.

Prof: Linhares e Júlio                                           Página 77
Na matriz                    , temos:




  Ou na matriz B = [ -1 0 2 5 ], temos: a11 = -1, a12 = 0, a13 = 2 e a14 = 5.

Denominações especiais
   Algumas matrizes, por suas características, recebem denominações
especiais.
      Matriz linha: matriz do tipo 1 x n, ou seja, com uma única linha. Por
   exemplo, a matriz A =[4 7 -3 1], do tipo 1 x 4.


         Matriz coluna: matriz do tipo m x 1, ou seja, com uma única coluna.



   Por            exemplo,      ,         do      tipo        3        x          1


      Matriz quadrada: matriz do tipo n x n, ou seja, com o mesmo
   número de linhas e colunas; dizemos que a matriz é de ordem n. Por

   exemplo, a matriz                é do tipo 2 x 2, isto é, quadrada de ordem
   2.
  Numa matriz quadrada definimos a diagonal principal e a diagonal
secundária. A principal é formada pelos elementos aij tais que i = j. Na
secundária, temos i + j = n + 1.

   Veja:




Prof: Linhares e Júlio                                                     Página 78
Observe a matriz a seguir:




a11 = -1 é elemento da diagonal principal, pis i = j = 1

a31= 5 é elemento da diagonal secundária, pois i + j = n + 1 ( 3 + 1 = 3 + 1)
       Matriz nula: matriz em que todos os elementos são nulos; é
   representada por 0m x n.


Por                          exemplo,                                        .


      Matriz diagonal: matriz quadrada em que todos os elementos que
   não estão na diagonal principal são nulos. Por exemplo:




       Matriz identidade: matriz quadrada em que todos os elementos da
   diagonal principal são iguais a 1 e os demais são nulos; é representada
   por In, sendo n a ordem da matriz. Por exemplo:



Prof: Linhares e Júlio                                               Página 79
Assim,         para      uma      matriz     identidade                                .


         Matriz transposta: matriz At obtida a partir da matriz A trocando-
     se ordenadamente as linhas por colunas ou as colunas por linhas. Por
     exemplo:




     Desse modo, se a matriz A é do tipo m x n, At é do tipo n x m.
  Note que a 1ª linha de A corresponde à 1ª coluna de At e a 2ª linha de A
corresponde à 2ª coluna de At.

        Matriz simétrica: matriz quadrada de ordem n tal que A = At . Por
     exemplo,




                         é simétrica, pois a12 = a21 = 5, a13 = a31 = 6, a23 = a32 =
4,       ou        seja,     temos     sempre           a      ij    =       a     ij.



        Matriz oposta: matriz -A obtida a partir de A trocando-se o sinal de

     todos os elementos de A. Por exemplo,                                         .


Prof: Linhares e Júlio                                                       Página 80
Igualdade de matrizes
  Duas matrizes, A e B, do mesmo tipo m x n, são iguais se, e somente se,
todos os elementos que ocupam a mesma posição são iguais:




                                               .



Operações envolvendo matrizes

Adição

   Dadas as matrizes                          , chamamos de soma dessas
matrizes a matriz               , tal que Cij = aij + bij , para todo
                         :


                                A+B=C


Exemplos:




Observação: A + B existe se, e somente se, A e B forem do mesmo tipo.

Propriedades
  Sendo A, B e C matrizes do mesmo tipo ( m x n), temos as seguintes
propriedades para a adição:

a) comutativa: A + B = B + A

b) associativa: ( A + B) + C = A + ( B + C)

Prof: Linhares e Júlio                                           Página 81
c) elemento neutro: A + 0 = 0 + A = A, sendo 0 a matriz nula m x n

d) elemento oposto: A + ( - A) = (-A) + A = 0

Subtração

  Dadas as matrizes                       , chamamos de diferença entre
essas matrizes a soma de A com a matriz oposta de B:


                             A-B=A+(-B)


Observe:




Multiplicação de um número real por uma matriz
  Dados um número real x e uma matriz A do tipo m x n, o produto de x
por A é uma matriz B do tipo m x n obtida pela multiplicação de cada
elemento de A por x, ou seja, bij = xaij:


                                      B = x.A


   Observe o seguinte exemplo:




Propriedades
  Sendo A e B matrizes do mesmo tipo ( m x n) e x e y números reais
quaisquer, valem as seguintes propriedades:
a) associativa: x . (yA) = (xy) . A



Prof: Linhares e Júlio                                               Página 82
b) distributiva de um número real em relação à adição de matrizes: x . (A +
B) = xA + xB
c) distributiva de uma matriz em relação à adição de dois números reais: (x
+ y) . A = xA + yA

d) elemento neutro : xA = A, para x=1, ou seja, A=A

Multiplicação de matrizes
  O produto de uma matriz por outra não é determinado por meio do
produto dos sus respectivos elementos.

  Assim, o produto das matrizes A = ( aij) m x p e B = ( bij) p x n é a matriz C
= (cij) m x n em que cada elemento cij é obtido por meio da soma dos
produtos dos elementos correspondentes da i-ésima linha de A pelos
elementos da j-ésima coluna B.


 Vamos multiplicar a matriz                             para entender como se
obtém cada Cij:
        1ª linha e 1ª coluna




        1ª linha e 2ª coluna




        2ª linha e 1ª coluna




        2ª linha e 2ª coluna



Prof: Linhares e Júlio                                                  Página 83
Assim,                 .

  Observe que:




  Portanto,      .A, ou seja, para a multiplicação de matrizes não vale a
propriedade comutativa.



  Vejamos outro exemplo com as matrizes                           :




  Da definição, temos que a matriz produto A . B só existe se o número de
colunas de A for igual ao número de linhas de B:




Prof: Linhares e Júlio                                           Página 84
A matriz produto terá o número de linhas de A (m) e o número de
colunas de B(n):
        Se A3 x 2 e B 2 x 5 , então ( A . B ) 3 x 5

        Se A 4 x 1 e B 2 x 3, então não existe o produto

        Se A         4   x   2   e B   2   x   1,     então ( A . B )   4   x   1



Propriedades
  Verificadas as condições de existência para a multiplicação de matrizes,
valem as seguintes propriedades:

a) associativa: ( A . B) . C = A . ( B . C )

b) distributiva em relação à adição: A . ( B + C ) = A . B + A . C ou
(A+B).C=A.C+B.C
c) elemento neutro: A . In = In . A = A, sendo In a matriz identidade de
ordem n

   Vimos que a propriedade comutativa, geralmente, não vale para a
multiplicação de matrizes. Não vale também o anulamento do produto, ou
seja: sendo 0 m x n uma matriz nula, A .B =0 m x n não implica,
necessariamente, que A = 0 m x n ou B = 0 m x n.


Matriz inversa
 Dada uma matriz A, quadrada, de ordem n, se existir uma matriz A', de
mesma ordem, tal que A . A' = A' . A = In , então A' é matriz inversa de A .
Representamos a matriz inversa por A-1 .




Prof: Linhares e Júlio                                                  Página 85
Grandezas - Introdução
    Entendemos por grandeza tudo aquilo que pode ser medido, contado.
As grandezas podem ter suas medidas aumentadas ou diminuídas.
    Alguns exemplos de grandeza: o volume, a massa, a superfície, o
comprimento, a capacidade, a velocidade, o tempo, o custo e a produção.

     É comum ao nosso dia-a-dia situações em que relacionamos duas ou
mais grandezas. Por exemplo:

     Em uma corrida de "quilômetros contra o relógio", quanto maior for a
velocidade, menor será o tempo gasto nessa prova. Aqui as grandezas são a
velocidade e o tempo.

    Num forno utilizado para a produção de ferro fundido comum, quanto
maior for o tempo de uso, maior será a produção de ferro. Nesse caso, as
grandezas são o tempo e a produção.

Grandezas diretamente proporcionais
    Um forno tem sua produção de ferro fundido de acordo com a tabela
abaixo:

                          Tempo
                                               Produção (Kg)
                          (minutos)
                          5                    100
                          10                   200
                          15                   300
                          20                   400

Observe que uma grandeza varia de acordo com a outra. Essas grandezas
são variáveis dependentes. Observe que:

Quando duplicamos              o      tempo,    a    produção   também   duplica.
5min ----> 100Kg
10 min ----> 200Kg
Quando triplicamos             o      tempo,    a    produção   também   triplica.
5min ----> 100Kg
15 min ----> 300Kg

Assim:



Prof: Linhares e Júlio                                                   Página 86
Duas grandezas variáveis dependentes são diretamente
       proporcionais quando a razão entre os valores da 1ª grandeza é
       igual a razão entre os valores correspondentes da 2ª


Verifique na tabela que a razão entre dois valores de uma grandeza é igual
a razão entre os dois valores correspondentes da outra grandeza.




Grandezas inversamente proporcionais
    Um ciclista faz um treino para a prova de "1000 metros contra o
relógio", mantendo em cada volta uma velocidade constante e obtendo,
assim, um tempo correspondente, conforme a tabela abaixo

                         Velocidade (m/s)   Tempo (s)
                         5                  200
                         8                  125
                         10                 100
                         16                 62,5
                         20                 50

Observe que uma grandeza varia de acordo com a outra. Essas grandezas
são variáveis dependentes. Observe que:

Quando duplicamos a velocidade, o tempo fica reduzido à metade.
5m/s ----> 200s
10 m/s ----> 100s
Quando quadriplicamos a velocidade, o tempo fica reduzido à quarta
parte.
5m/s ----> 200s
20 m/s ----> 50s

Assim:




Prof: Linhares e Júlio                                             Página 87
Duas grandezas variáveis dependentes são inversamente proporcionais quando a
razão entre os valores da 1ª grandeza é igual ao inverso da razão entre os
valores correspondentes da 2ª.


     Verifique na tabela que a razão entre dois valores de uma grandeza é igual
     ao inverso da razão entre os dois valores correspondentes da outra
     grandeza.




     Prof: Linhares e Júlio                                            Página 88
POLINÔMIOS
     Definição

   Uma função polinomial ou simplesmente polinômio, é toda função
definida pela relação P(x)=anxn + an-1.xn-1 + an-2.xn-2 + ... + a2x2 + a1x + a0.
   Onde:
   an, an-1, an-2, ..., a2, a1, a0 são números reais chamados coeficientes.
   n  IN
   x  C (nos complexos) é a variável.

    GRAU DE UM POLINÔMIO:

   Grau de um polinômio é o expoente máximo que ele possui. Se o
coeficiente an0, então o expoente máximo n é dito grau do polinômio e
indicamos gr(P)=n. Exemplos:
   a) P(x)=5 ou P(x)=5.x0 é um polinômio constante, ou seja, gr(P)=0.
   b) P(x)=3x+5 é um polinômio do 1º grau, isto é, gr(P)=1.
   c) P(x)=4x5+7x4 é um polinômio do 5º grau, ou seja, gr(P)=5.

    Obs: Se P(x)=0, não se define o grau do polinômio.


     Valor numérico

    O valor numérico de um polinômio P(x) para x=a, é o número que se
obtém substituindo x por a e efetuando todas as operações indicadas pela
relação que define o polinômio. Exemplo:
    Se P(x)=x3+2x2+x-4, o valor numérico de P(x), para x=2, é:
    P(x)= x3+2x2+x-4
    P(2)= 23+2.22+2-4
    P(2)= 14

   Observação: Se P(a)=0, o número a chamado raiz ou zero de P(x).
   Por exemplo, no polinômio P(x)=x2-3x+2 temos P(1)=0; logo, 1 é raiz
ou zero desse polinômio.




Prof: Linhares e Júlio                                                  Página 89
Alguns exercícios resolvidos:

    1º) Sabendo-se que –3 é raiz de P(x)=x3+4x2-ax+1, calcular o valor de a.
    Resolução: Se –3 é raiz de P(x), então P(-3)=0.
    P(-3)=0 => (-3)3+4(-3)2-a.(-3)+1 = 0
    3a = -10 => a=-10/3
    Resposta: a=-10/3

    2º) Calcular m  IR para que o polinômio
    P(x)=(m2-1)x3+(m+1)x2-x+4 seja:
    a) do 3ºgrau         b) do 2º grau             c) do 1º grau

    Resposta:
    a) para o polinômio ser do 3º grau, os coeficientes de x2 e x3 devem ser
       diferentes de zero. Então:
       m2-10 => m21 => m1
       m+10 => m-1
       Portanto, o polinômio é do 3º grau se m1 e m-1.

    b) para o polinômio ser do 2º grau, o coeficiente de x 3 deve ser igual a
       zero e o coeficiente de x2 diferente de zero. Então:
       m2-1=0 => m2=1 => m=1
       m+10 => m-1
       Portanto, o polinômio é do 2º grau se m=1.

    c) para o polinômio ser do 1º grau, os coeficientes de x2 e x3 devem ser
       iguais a zero. Então:
       m2-1=0 => m2=1 => m=1
       m+1=0 => m=-1
       Portanto, o polinômio é do 1º grau se m=-1.

3º) Num polinômio P(x), do 3º grau, o coeficiente de x3 é 1. Se
P(1)=P(2)=0 e P(3)=30, calcule o valor de P(-1).
Resolução:
Temos o polinômio: P(x)=x3+ax2+bx+c.
Precisamos encontrar os valores de a,b e c (coeficientes).
Vamos utilizar os dados fornecidos pelo enunciado do problema:


P(1)=0 => (1)3+a.(1)2+b(1)+c = 0 => 1+a+b+c=0 => a+b+c=-1
P(2)=0 => (2)3+a.(2)2+b(2)+c = 0 => 8+4a+2b+c=0 => 4a+2b+c=-8
P(3)=30 => (3)3+a.(3)2+b(3)+c = 30 => 27+9a+3b+c=30 => 9a+3b+c=3

Prof: Linhares e Júlio                                               Página 90
Temos um sistema de três variáveis:
  a  b  c  -1
  
  4a  2b  c  -8
  9a  3b  c  3
  

Resolvendo esse sistema encontramos as soluções:
a=9, b=-34, c=24
Portanto o polinômio em questão é P(x)= x3+9x2-34x+24.
O problema pede P(-1):
P(-1)= (-1)3+9(-1)2-34(-1)+24 => P(-1)=-1+9+34+24
P(-1)= 66
Resposta: P(-1)= 66


     Polinômios iguais

   Dizemos que dois polinômios A(x) e B(x) são iguais ou idênticos (e
indicamos A(x)B(x)) quando assumem valores numéricos iguais para
qualquer valor comum atribuído à variável x. A condição para que dois
polinômios sejam iguais ou idênticos é que os coeficientes dos termos
correspondentes sejam iguais.
   Exemplo:
   Calcular a,b e c, sabendo-se que x2-2x+1  a(x2+x+1)+(bx+c)(x+1).
   Resolução: Eliminando os parênteses e somando os termos semelhantes
do segundo membro temos:
   x2-2x+1  ax2+ax+a+bx2+bx+cx+c
   1x2-2x+1  (a+b)x2+(a+b+c)x+(a+c)
   Agora igualamos os coeficientes correspondentes:
           a  b  1
           
           a  b  c  2
           a  c  1
           


    Substituindo a 1ª equação na 2ª:
    1+c = -2 => c=-3.
    Colocando esse valor de c na 3ª equação, temos:
    a-3=1 => a=4.
    Colocando esse valor de a na 1ª equação, temos:
    4+b=1 => b=-3.
    Resposta: a=4, b=-3 e c=-3.

Prof: Linhares e Júlio                                         Página 91
Obs: um polinômio é dito identicamente nulo se tem todos os seus
coeficientes nulos.


     Divisão de polinômios

   Sejam dois polinômios P(x) e D(x), com D(x) não nulo.
   Efetuar a divisão de P por D é determinar dois polinômios Q(x) e R(x),
que satisfaçam as duas condições abaixo:
   1ª) Q(x).D(x) + R(x) = P(x)
   2ª) gr(R) < gr(D) ou R(x)=0
                                     P( x)    D( x )
                                     R( x)    Q( x)

    Nessa divisão:
    P(x) é o dividendo.
    D(x) é o divisor.
    Q(x) é o quociente.
    R(x) é o resto da divisão.

   Obs: Quando temos R(x)=0 dizemos que a divisão é exata, ou seja, P(x)
é divisível por D(x) ou D(x) é divisor de P(x).

                         Se D(x) é divisor de P(x)  R(x)=0


Exemplo:
Determinar o quociente de P(x)=x4+x3-7x2+9x-1 por D(x)=x2+3x-2.
Resolução: Aplicando o método da chave, temos:

                 x 4  x3  7 x 2  9 x  1            x 2  3x  2
                x 4  3x3  2 x 2                 x 2  2 x  1  Q( x)
                    2 x3  5 x 2  9 x  1
                    2 x3  6 x 2  4 x
                              x2  5x  1
                            x 2  3x  2
                                      2 x  1  R( x)



Prof: Linhares e Júlio                                                     Página 92
Verificamos que:

        x 4  -  1  (x 2  3x - 2) (x 2 - 2x  1)  (2x  1)
          x 7x  9x   
              3   2
                      -
                                                            
                   P(x)                    D(x)            Q(x)   R(x)




     Divisão de um polinômio por um binômio da forma ax+b

    Vamos calcular o resto da divisão de P(x)=4x2-2x+3 por D(x)=2x-1.
    Utilizando o método da chave temos:

                           4x2  2x  3           2x  1
                           4x2  2x              2x
                                       3

    Logo: R(x)=3
    A raiz do divisor é 2x-1=0 => x=1/2.
    Agora calculamos P(x) para x=1/2.
    P(1/2) = 4(1/4) – 2(1/2) + 3
    P(1/2) = 3

   Observe que R(x) = 3 = P(1/2)
   Portanto, mostramos que o resto da divisão de P(x) por D(x) é igual ao
valor numérico de P(x) para x=1/2, isto é, a raiz do divisor.



     Teorema do resto

O resto da divisão de um polinômio P(x) pelo binômio ax+b é igual a P(-b/a).

    Note que –b/a é a raiz do divisor.

    Exemplo: Calcule o resto da divisão de x2+5x-1 por x+1.
    Resolução: Achamos a raiz do divisor:
    x+1=0 => x=-1
    Pelo teorema do resto sabemos que o resto é igual a P(-1):
    P(-1)=(-1)2+5.(-1)-1 => P(-1) = -5 = R(x)
    Resposta: R(x) = -5.


Prof: Linhares e Júlio                                                   Página 93
 Teorema de D’Alembert

         Um polinômio P(x) é divisível pelo binômio ax+b se P(-b/a)=0


   Exemplo: Determinar o valor de p, para que o polinômio P(x)=2x3+5x2-
px+2 seja divisível por x-2.
   Resolução: Se P(x) é divisível por x-2, então P(2)=0.
      P(2)=0 => 2.8+5.4-2p+2=0 => 16+20-2p+2=0 => p=19
   Resposta: p=19.


     Divisão de um polinômio pelo produto (x-a)(x-b)

   Vamos resolver o seguinte problema: calcular o resto da divisão do
polinômio P(x) pelo produto (x-a)(x-b), sabendo-se que os restos da divisão
de P(x) por (x-a) e por (x-b) são, respectivamente, r1 e r2.
   Temos:
   a é a raiz do divisor x-a, portanto P(a)=r1      (eq. 1)
   b é a raiz do divisor x-b, portanto P(b)=r2      (eq. 2)
   E para o divisor (x-a)(x-b) temos P(x)=(x-a)(x-b) Q(x) + R(x)       (eq.
3)

   O resto da divisão de P(x) por (x-a)(x-b) é no máximo do 1º grau, pois o
divisor é do 2º grau; logo:
   R(x)=cx+d

    Da eq.3 vem:
    P(x)=(x-a)(x-b) Q(x) + cx + d
    Fazendo:
    x=a => P(a) = c(a)+d        (eq. 4)
    x=b => P(b) = c(b)+d        (eq. 5)

    Das equações 1, 2, 4 e 5 temos:

               ca  d  r1
               
               cb  d  r2

    Resolvendo o sistema obtemos:



Prof: Linhares e Júlio                                             Página 94
r1  r2        ar  ar1
      c          e d 2         , com a  b
          ab             ab
                     r r     ar  ar1
      Logo : R( x)  1 2 x  2         , com a  b
                      ab       ab


    Observações:
    1ª) Se P(x) for divisível por (x-a) e por (x-b), temos:
    P(a)= r1 =0
    P(b)= r2 =0
    Portanto, P(x) é divisível pelo produto (x-a)(x-b), pois:

                    r1  r2   ar  ar1
         R( x)             x 2        00  0
                     a b       a b

   2ª) Generalizando, temos:
      Se P(x) é divisível por n fatores distintos (x-a1), (x-a2),..., (x-an) então
P(x) é divisível pelo produto (x-a1)(x-a2)...(x-an).

   Exemplo:
   Um polinômio P(x) dividido por x dá resto 6 e dividido por (x-1) dá
resto 8. Qual o resto da divisão de P(x) por x(x-1)?
   Resolução:
   0 é a raiz do divisor x, portanto P(0)=6       (eq. 1)

    1 é a raiz do divisor x-1, portanto P(1)=8   (eq. 2)
    E para o divisor x(x-1) temos P(x)=x(x-1) Q(x) + R(x)            (eq. 3)

   O resto da divisão de P(x) por x(x-1) é no máximo do 1º grau, pois o
divisor é do 2º grau; logo:
   R(x)=ax+b

    Da eq.3 vem:
    P(x)=x(x-1) Q(x) + ax + b
    Fazendo:
    x=0 => P(0) = a(0)+b => P(0) = b              (eq. 4)
    x=1 => P(1) = a(1)+b => P(1) = a+b            (eq. 5)

    Das equações 1, 2, 4 e 5 temos:

               b  6
               
               a  b  8
Prof: Linhares e Júlio                                                   Página 95
Logo, b=6 e a=2.
    Agora achamos o resto: R(x) = ax+b = 2x+6
    Resposta: R(x) = 2x+6.


     O dispositivo de Briot-Ruffini

   Serve para efetuar a divisão de um polinômio P(x) por um binômio da
forma (ax+b).
   Exemplo: Determinar o quociente e o resto da divisão do polinômio
P(x)=3x3-5x2+x-2 por (x-2).
   Resolução:

                  
                   
               RAIZ DO DIVISOR   ES DE P(x) 
                                       COEFICIENT
                                                     
                   2             3    5          1     2
                                          3.(2)  5     1.(2)  1   3.(2)  2

                                 1 
                                 3        3
                                                                    4
                                                                     
                                  COEFICIENTES DO QUOCIENTE Q(x)     RESTO




   Observe que o grau de Q(x) é uma unidade inferior ao de P(x), pois o
divisor é de grau 1.
   Resposta: Q(x)=3x2+x+3 e R(x)=4.

   Para a resolução desse problema seguimos os seguintes passos:
   1º) Colocamos a raiz do divisor e os coeficientes do dividendo
ordenadamente na parte de cima da “cerquinha”.
   2º) O primeiro coeficiente do dividendo é repetido abaixo.
   3º) Multiplicamos a raiz do divisor por esse coeficiente repetido abaixo
e somamos o produto com o 2º coeficiente do dividendo, colocando o
resultado abaixo deste.
   4º) Multiplicamos a raiz do divisor pelo número colocado abaixo do 2º
coeficiente e somamos o produto com o 3º coeficiente, colocando o
resultado abaixo deste, e assim sucessivamente.
   5º) Separamos o último número formado, que é igual ao resto da
divisão, e os números que ficam à esquerda deste serão os coeficientes do
quociente.




Prof: Linhares e Júlio                                                           Página 96
 Decomposição de um polinômio em fatores

   Vamos analisar dois casos:
   1º caso: O polinômio é do 2º grau.
      De uma forma geral, o polinômio de 2º grau P(x)=ax2+bx+c que
admite as raízes r1 e r2 pode ser decomposto em fatores do 1º grau, da
seguinte forma:

                         ax2+bx+c = a(x-r1)(x-r2)

        Exemplos:
        1) Fatorar o polinômio P(x)=x2-4.
           Resolução: Fazendo x2-4=0, obtemos as raízes r1=2 e r2=-2.
           Logo: x2-4 = (x-2)(x+2).

        2) Fatorar o polinômio P(x)=x2-7x+10.
           Resolução: Fazendo x2-7x+10=0, obtemos as raízes r1=5 e r2=2.
           Logo: x2-7x+10 = (x-5)(x-2).


   2º caso: O polinômio é de grau maior ou igual a 3.
      Conhecendo uma das raízes de um polinômio de 3º grau, podemos
decompô-lo num produto de um polinômio do 1º grau por um polinômio do
2º grau e, se este tiver raízes, podemos em seguida decompô-lo também.

    Exemplo: Decompor em fatores do 1º grau o polinômio 2x3-x2-x.
    Resolução:
      2x3-x2-x = x.(2x2-x-1)  colocando x em evidência
      Fazendo x.(2x2-x-1) = 0 obtemos: x=0 ou 2x2-x-1=0.
      Uma das raízes já encontramos (x=0).
      As outras duas saem da equação: 2x2-x-1=0 => r1=1 e r2=-1/2.
      Portanto, o polinômio 2x3-x2-x, na forma fatorada é:
      2.x.(x-1).(x+(1/2)).

Generalizando, se o polinômio P(x)=anxn+an-1xn-1+...+a1x+a0 admite n
raízes r1, r2,..., rn, podemos decompô-lo em fatores da seguinte forma:

              anxn+an-1xn-1+...+a1x+a0 = an(x-r1)(x-r2)...(x-rn)




Prof: Linhares e Júlio                                             Página 97
Observações:
     1) Se duas, três ou mais raiz forem iguais, dizemos que são raízes
        duplas, triplas, etc.
     2) Uma raiz r1 do polinômio P(x) é dita raiz dupla ou de
        multiplicidade 2 se P(x) é divisível por (x-r1)2 e não por (x-r1)3.




Prof: Linhares e Júlio                                             Página 98
PROBABILIDADE
   A história da teoria das probabilidades, teve início com os jogos de
cartas, dados e de roleta. Esse é o motivo da grande existência de exemplos
de jogos de azar no estudo da probabilidade. A teoria da probabilidade
permite que se calcule a chance de ocorrência de um número em um
experimento aleatório.

   Experimento Aleatório

   É aquele experimento que quando repetido em iguais condições, podem
fornecer resultados diferentes, ou seja, são resultados explicados ao acaso.
Quando se fala de tempo e possibilidades de ganho na loteria, a abordagem
envolve cálculo de experimento aleatório.

   Espaço Amostral

   É o conjunto de todos os resultados possíveis de um experimento
aleatório. A letra que representa o espaço amostral, é S.

   Exemplo:
  Lançando uma moeda e um dado, simultaneamente, sendo S o espaço
amostral, constituído pelos 12 elementos:
   S = {K1, K2, K3, K4, K5, K6, R1, R2, R3, R4, R5, R6}

    1. Escreva explicitamente os seguintes eventos: A={caras e m número
       par aparece}, B={um número primo aparece}, C={coroas e um
       número ímpar aparecem}.
    2. Idem, o evento em que:

             a)      A ou B ocorrem;

             b)      B e C ocorrem;
             c)      Somente B ocorre.

    3. Quais dos eventos A,B e C são mutuamente exclusivos



Resolução:

    1. Para obter A, escolhemos os elementos de S constituídos de um K e
       um número par: A={K2, K4, K6};

Prof: Linhares e Júlio                                              Página 99
Para obter B, escolhemos os pontos de S constituídos de números
         primos: B={K2,K3,K5,R2,R3,R5}
         Para obter C, escolhemos os pontos de S constituídos de um R e um
         número ímpar: C={R1,R3,R5}.

    2. (a) A ou B = AUB = {K2,K4,K6,K3,K5,R2,R3,R5}

        (b) B e C = B  C = {R3,R5}

        (c) Escolhemos os elementos de B que não estão em A ou C;

        B  Ac  Cc = {K3,K5,R2}

    3. A e C são mutuamente exclusivos, porque A  C = 
       

Conceito de probabilidade
    Se em um fenômeno aleatório as possibilidades são igualmente
    prováveis, então a probabilidade de ocorrer um evento A é:




    Por, exemplo, no lançamento de um dado, um número par pode ocorrer
    de 3 maneiras diferentes dentre 6 igualmente prováveis, portanto, P =
    3/6= 1/2 = 50%
    Dizemos que um espaço amostral S (finito) é equiprovável quando seus
    eventos elementares têm probabilidades iguais de ocorrência.

    Num espaço amostral equiprovável S (finito), a probabilidade de
    ocorrência de um evento A é sempre:




Prof: Linhares e Júlio                                              Página 100
Propriedades Importantes:
    1. Se A e A’ são eventos complementares, então:
    P( A ) + P( A' ) = 1

    2. A probabilidade de um evento é sempre um número entre 
    (probabilidade de evento impossível) e 1 (probabilidade do evento
    certo).




    Probabilidade Condicional
     Antes da realização de um experimento, é necessário que já tenha
    alguma informação sobre o evento que se deseja observar. Nesse caso, o
    espaço amostral se modifica e o evento tem a sua probabilidade de
    ocorrência alterada.

     Fórmula de Probabilidade Condicional
    P(E1 e E2 e E3 e ...e En-1 e En) é igual a P(E1).P(E2/E1).P(E3/E1 e
    E2)...P(En/E1 e E2 e ...En-1).

    Onde P(E2/E1) é a probabilidade de ocorrer E2, condicionada pelo fato
    de já ter ocorrido E1;

    P(E3/E1 e E2) é a probabilidade ocorrer E3, condicionada pelo fato de já
    terem ocorrido E1 e E2;

P(Pn/E1 e E2 e ...En-1) é a probabilidade de ocorrer En, condicionada ao fato
de já ter ocorrido E1 e E2...En-1.



   Exemplo:
   Uma urna tem 30 bolas, sendo 10 vermelhas e 20 azuis. Se ocorrer um
sorteio de 2 bolas, uma de cada vez e sem reposição, qual será a
probabilidade de a primeira ser vermelha e a segunda ser azul?

   Resolução:
   Seja o espaço amostral S=30 bolas, bolinhas e considerarmos os
seguintes eventos:

Prof: Linhares e Júlio                                              Página 101
Apostila Professor Linhares
Apostila Professor Linhares
Apostila Professor Linhares
Apostila Professor Linhares
Apostila Professor Linhares
Apostila Professor Linhares
Apostila Professor Linhares
Apostila Professor Linhares
Apostila Professor Linhares
Apostila Professor Linhares
Apostila Professor Linhares
Apostila Professor Linhares
Apostila Professor Linhares
Apostila Professor Linhares
Apostila Professor Linhares
Apostila Professor Linhares
Apostila Professor Linhares
Apostila Professor Linhares
Apostila Professor Linhares
Apostila Professor Linhares
Apostila Professor Linhares
Apostila Professor Linhares
Apostila Professor Linhares
Apostila Professor Linhares
Apostila Professor Linhares
Apostila Professor Linhares
Apostila Professor Linhares
Apostila Professor Linhares
Apostila Professor Linhares
Apostila Professor Linhares
Apostila Professor Linhares
Apostila Professor Linhares
Apostila Professor Linhares
Apostila Professor Linhares
Apostila Professor Linhares
Apostila Professor Linhares
Apostila Professor Linhares
Apostila Professor Linhares
Apostila Professor Linhares

Mais conteúdo relacionado

Mais procurados

Mais procurados (19)

Matemática - Análise Combinatória - Com Exercícios Resolvidos - www.CentroApo...
Matemática - Análise Combinatória - Com Exercícios Resolvidos - www.CentroApo...Matemática - Análise Combinatória - Com Exercícios Resolvidos - www.CentroApo...
Matemática - Análise Combinatória - Com Exercícios Resolvidos - www.CentroApo...
 
Análise combinatória (resumo e exercícios)
Análise combinatória (resumo e exercícios)Análise combinatória (resumo e exercícios)
Análise combinatória (resumo e exercícios)
 
Análise combinatória
Análise  combinatóriaAnálise  combinatória
Análise combinatória
 
Análise Combinatória
Análise CombinatóriaAnálise Combinatória
Análise Combinatória
 
Combinatoria
CombinatoriaCombinatoria
Combinatoria
 
Aula 01 análise combinatória
Aula 01   análise combinatóriaAula 01   análise combinatória
Aula 01 análise combinatória
 
Gradmat combinatoria
Gradmat combinatoriaGradmat combinatoria
Gradmat combinatoria
 
Preparação para a Prova Final
Preparação para a Prova FinalPreparação para a Prova Final
Preparação para a Prova Final
 
Raciocinio logico aula 2
Raciocinio logico aula 2Raciocinio logico aula 2
Raciocinio logico aula 2
 
OBMEP-2010-Solução da Prova
OBMEP-2010-Solução da ProvaOBMEP-2010-Solução da Prova
OBMEP-2010-Solução da Prova
 
Raciocinio logico
Raciocinio logicoRaciocinio logico
Raciocinio logico
 
Exercicios de revisão 7 ano
Exercicios de revisão 7 anoExercicios de revisão 7 ano
Exercicios de revisão 7 ano
 
Apostila de análise combinatória
Apostila de análise combinatóriaApostila de análise combinatória
Apostila de análise combinatória
 
15032014
1503201415032014
15032014
 
Exercícios Extras
Exercícios ExtrasExercícios Extras
Exercícios Extras
 
Orientação Espacial
Orientação EspacialOrientação Espacial
Orientação Espacial
 
Problemas
ProblemasProblemas
Problemas
 
Prova Canguru da Matemática - 9º ano - 2017
Prova Canguru da Matemática - 9º ano - 2017Prova Canguru da Matemática - 9º ano - 2017
Prova Canguru da Matemática - 9º ano - 2017
 
Apostila bb ze_moreira_
Apostila bb ze_moreira_Apostila bb ze_moreira_
Apostila bb ze_moreira_
 

Semelhante a Apostila Professor Linhares

Análise combinatória (resumo)
Análise combinatória (resumo)Análise combinatória (resumo)
Análise combinatória (resumo)josivaldopassos
 
Análise combinatória (resumo)
Análise combinatória (resumo)Análise combinatória (resumo)
Análise combinatória (resumo)josivaldopassos
 
Análise combinatória (resumo e exercícios)
Análise combinatória (resumo e exercícios)Análise combinatória (resumo e exercícios)
Análise combinatória (resumo e exercícios)josivaldopassos
 
Analise combinatoria 1
Analise combinatoria 1Analise combinatoria 1
Analise combinatoria 1SEDUC-PA
 
Combinatória.pptxCombinatória.pptxCombinatória.pptxCombinatória.pptx
Combinatória.pptxCombinatória.pptxCombinatória.pptxCombinatória.pptxCombinatória.pptxCombinatória.pptxCombinatória.pptxCombinatória.pptx
Combinatória.pptxCombinatória.pptxCombinatória.pptxCombinatória.pptxalessandraoliveira324
 
Caderno matematica
Caderno matematicaCaderno matematica
Caderno matematicafazag
 
Análise combinatória (resumo)
Análise combinatória (resumo)Análise combinatória (resumo)
Análise combinatória (resumo)josivaldopassos
 
Análise combinatória (resumo)
Análise combinatória (resumo)Análise combinatória (resumo)
Análise combinatória (resumo)josivaldopassos
 
Gabarito 1ª Fase - Nível 1 - 2012
Gabarito 1ª Fase - Nível 1 - 2012Gabarito 1ª Fase - Nível 1 - 2012
Gabarito 1ª Fase - Nível 1 - 2012oim_matematica
 
Implementação modulo3
Implementação modulo3Implementação modulo3
Implementação modulo3inechidias
 
Implementação modulo3
Implementação modulo3Implementação modulo3
Implementação modulo3inechidias
 
Implementação modulo3
Implementação modulo3Implementação modulo3
Implementação modulo3inechidias
 
Implementação modulo3
Implementação modulo3Implementação modulo3
Implementação modulo3inechidias
 
Implementação modulo3
Implementação modulo3Implementação modulo3
Implementação modulo3inechidias
 
Implementação modulo3
Implementação modulo3Implementação modulo3
Implementação modulo3inechidias
 
Implementação modulo3
Implementação modulo3Implementação modulo3
Implementação modulo3inechidias
 
Implementação modulo3
Implementação modulo3Implementação modulo3
Implementação modulo3inechidias
 
Questao es sa__lista_final (1)
Questao es sa__lista_final (1)Questao es sa__lista_final (1)
Questao es sa__lista_final (1)Jorge Pedro
 
Aula 4 pfc 24-04-2021
Aula 4 pfc 24-04-2021Aula 4 pfc 24-04-2021
Aula 4 pfc 24-04-2021SEDUC-PA
 

Semelhante a Apostila Professor Linhares (20)

ANÁLISE COMBINATÓRIA
ANÁLISE COMBINATÓRIA ANÁLISE COMBINATÓRIA
ANÁLISE COMBINATÓRIA
 
Análise combinatória (resumo)
Análise combinatória (resumo)Análise combinatória (resumo)
Análise combinatória (resumo)
 
Análise combinatória (resumo)
Análise combinatória (resumo)Análise combinatória (resumo)
Análise combinatória (resumo)
 
Análise combinatória (resumo e exercícios)
Análise combinatória (resumo e exercícios)Análise combinatória (resumo e exercícios)
Análise combinatória (resumo e exercícios)
 
Analise combinatoria 1
Analise combinatoria 1Analise combinatoria 1
Analise combinatoria 1
 
Combinatória.pptxCombinatória.pptxCombinatória.pptxCombinatória.pptx
Combinatória.pptxCombinatória.pptxCombinatória.pptxCombinatória.pptxCombinatória.pptxCombinatória.pptxCombinatória.pptxCombinatória.pptx
Combinatória.pptxCombinatória.pptxCombinatória.pptxCombinatória.pptx
 
Caderno matematica
Caderno matematicaCaderno matematica
Caderno matematica
 
Análise combinatória (resumo)
Análise combinatória (resumo)Análise combinatória (resumo)
Análise combinatória (resumo)
 
Análise combinatória (resumo)
Análise combinatória (resumo)Análise combinatória (resumo)
Análise combinatória (resumo)
 
Gabarito 1ª Fase - Nível 1 - 2012
Gabarito 1ª Fase - Nível 1 - 2012Gabarito 1ª Fase - Nível 1 - 2012
Gabarito 1ª Fase - Nível 1 - 2012
 
Implementação modulo3
Implementação modulo3Implementação modulo3
Implementação modulo3
 
Implementação modulo3
Implementação modulo3Implementação modulo3
Implementação modulo3
 
Implementação modulo3
Implementação modulo3Implementação modulo3
Implementação modulo3
 
Implementação modulo3
Implementação modulo3Implementação modulo3
Implementação modulo3
 
Implementação modulo3
Implementação modulo3Implementação modulo3
Implementação modulo3
 
Implementação modulo3
Implementação modulo3Implementação modulo3
Implementação modulo3
 
Implementação modulo3
Implementação modulo3Implementação modulo3
Implementação modulo3
 
Implementação modulo3
Implementação modulo3Implementação modulo3
Implementação modulo3
 
Questao es sa__lista_final (1)
Questao es sa__lista_final (1)Questao es sa__lista_final (1)
Questao es sa__lista_final (1)
 
Aula 4 pfc 24-04-2021
Aula 4 pfc 24-04-2021Aula 4 pfc 24-04-2021
Aula 4 pfc 24-04-2021
 

Apostila Professor Linhares

  • 1. Apostila de Matemática Professor: Linhares e Júlio. [...] a Matemática procura compreender os modelos que permeiam o mundo que nos rodeia assim como a mente dentro de nós. […] Assim é necessário enfatizar: — a procurar de soluções, e não apenas a memorização de procedimentos; — a exploração de modelos, e não apenas a memorização de fórmulas; — a formulação de conjecturas, e não apenas a resolução de exercícios. [...] com essas ênfases, os estudantes terão a oportunidade de estudar a Matemática como uma disciplina exploradora, dinâmica, que se desenvolve, em lugar de ser uma disciplina que tem um corpo rígido, absoluto, fechado, cheio de regras que precisam ser memorizadas. Shoenfeld (1992). 26/01/2011
  • 2. Análise Combinatória Fatorial de um número: n!=n.(n-1).(n-2)...3.2.1 Definições especiais: 0!=1 1!=1 100!101! 1) Calcule o valor da expressão . 99! 100!101! 100.99!101.100.99!   100  101.100  100  10100  10200 99! 99! ( x  1)! 2) Resolva a equação  56. ( x  1)! ( x  1)! ( x  1)( x)( x  1)!  56   56  ( x  1)( x)  56  x 2  x  56  ( x  1)! ( x  1)!  1  225  1  15 x  7  x 2  x  56  0  x   x  2 2 x  -8 Resposta : x  7, pois não existe fatorial de um número negativo. 3) Quatro times de futebol (Grêmio, Santos, São Paulo e Flamengo) disputam o torneio dos campeões do mundo. Quantas são as possibilidades para os três primeiros lugares? R : Existem 4 possibilidades para o 1º lugar, sobrando 3 possibilidades para o 2º lugar e 2 possibilidades para o 3º lugar  4.3.2  24 possibilidades. Arranjo simples: n! An, p  (n  p)! A6, 2  A4,3  A5, 2 4) Calcule . A9, 2  A8,1 6! 4! 5!   A6, 2  A4,3  A5, 2 (6  2)! (4  3)! (5  2)! 30  24  20 34 17     A9, 2  A8,1 9! 8! 72  8 80 40  (9  2)! (8  1)! Prof: Linhares e Júlio Página 2
  • 3. 5) Quantos números de 3 algarismos distintos podemos formar com o algarismos do sistema decimal (0,1,2,3,4,5,6,7,8,9) sem os repetir, de modo que : a) COM ECEM COM 1. R : O número pode possuir três algarismos , sendo que para o primeiro existe apenas 1 possibilidade (1) e para os outros dois ainda existem 9 números disponíveis : 9! 9! 9.8.7! 1. A9, 2     9.8  72 números. (9  2)! 7! 7! b) COM ECEM COM 2 E TERM INEM COM 5. R : Para o primeiro algarismo existe apenas 1 possibilidade (2), e para o terceiro também existe apenas 1 possibilidade (5). Para o segundo ainda existem 8 possibilidades : 8! 8! 8.7! 1.1. A8,1     8 números. (8  1)! 7! 7! c) SEJAM DIVISÍVEIS POR 5. R : Para um número ser divisível 5, ele deve terminar com 0 ou com 5. Primeirame nte vamos calcular o número de divisíveis por 5 que terminam com 0 :  Para o terceiro algarismo existe apenas 1 possibilidade (0), e para os dois primeiros ainda existem 9 números disponíveis. Portanto o número de divisíveis por 5 que terminam com 0 é : 9! 9! 9.8.7! 1. A9, 2     9.8  72 números. (9  2)! 7! 7!  Agora calculamos quantos divisíveis por 5 terminam com 5 : para o terceiro algarismo existe apenas uma possibilidade (5). Para o primeiro algarismo existem ainda 8 possibilidades, pois o número não pode começar com 0 (senão seria um número de 2 algarismos ). E para o segundo algarismo também existem 8 possibilidades (o segundo algarismo pode ser 0). 8! 8! 8! 8! 8.7! 8.7! 1. A8,1 . A8,1  .  .  .  8.8  64 números. (8  1)! (8  1)! 7! 7! 7! 7! Resposta : O número de divisíveis por 5 é 72  64  136 números. 6) Quantos são os números compreendidos entre 2000 e 3000 formados por algarismos distintos escolhidos entre 1,2,3,4,5,6,7,8 e 9? R : O número deve ter quatro algarismos (pois está entre 2000 e 3000). Para o primeiro algarismo existe apenas uma possibilidade (2), e para os outros três ainda existem 8 números disponíveis, então : 8! 8! 8.7.6.5! 1. A8,3     8.7.6  336 números. (8  3)! 5! 5! Prof: Linhares e Júlio Página 3
  • 4. Permutação Simples: É um caso particular de arranjo simples. É o tipo de agrupamento ordenado onde entram todos os elementos. Pn  n! 7) Quantos números de 5 algarismos distintos podem ser formados por 1,2,3,5 e 8? P5  5! 5.4.3.2.1  120 números. 8) Quantos anagramas da palavra EDITORA : a) COM EÇAM POR A. Para a primeira letra existe apenas uma possibilidade (A), e para as outras 6 letras existem 6 possibilidades. Então o total é : 1.P6  1.6! 6.5.4.3.2.1  720 anagramas. b) COM EÇAM POR A e terminam com E. Para a primeira letra existe 1 possibilidade (A), e para última também só existe 1 (E), e para as outras 5 letras existem 5 possibilidades. Então o total é : 1.1.P5  1.1.5! 5.4.3.2.1  120 anagramas. 8) Calcule de quantas maneiras podem ser dipostas 4 damas e 4 cavalheiro s, numa fila, de forma que não fiquem juntos dois cavalheiro s e duas damas. R :Existem duas maneiras de fazer isso : C - D - C - D - C - D - C - D ou D - C - D - C - D - C - D - C Colocando um cavalheiro na primeira posição temos como número total de maneiras : P4 .P4  4!.4! 24.24  576 maneiras. Colocando uma dama na primeira posição temos também : P4 .P4  4!.4! 24.24  576 maneiras. Portanto o total é 576  576  1152 maneiras. Combinação Simples: é o tipo de agrupamento em que um grupo difere do outro apenas pela natureza dos elementos componentes. n! Cn, p  p!(n  p)! Prof: Linhares e Júlio Página 4
  • 5. 9) Resolver a equação C m ,3  C m , 2  0. m! m!  0 3!(m  3)! 2!(m  2)! m.(m  1).(m  2).(m  3)! m.(m  1).(m  2)!  0 3!(m  3)! 2!(m  2)! m.(m  1).(m  2) m.(m  1)  0 3! 2! m 3  2m 2  m 2  2m m 2  m  0 6 2 m 3  3m 2  2m  3m 2  3m  0  m 3  6 m 2  5m  0 6 6  16 m '  5 m 2  6m  5  0  m    2 m ' '  1 Resposta : m  5. obs : m  1 não é a resposta porque não pode haver C1,3. 10) Com 10 espécies de frutas, quantos tipos de salada, contendo 6 espécies diferentes podem ser feitas? 10! 10.9.8.7.6! 5040 5040 C10, 6      210 tipos de saladas. 6!.(10  6)! 6!.4! 4! 24 11) Numa reunião com 7 rapazes e 6 moças, quantas comissões podemos formar com 3 rapazes e 4 moças? RAPAZES- C 7 ,3 M OÇAS- C 6, 4 O resultado é o produto C 7 ,3 .C 6, 4 . 7! 6! 7.6.5.4! 6.5.4! 210 30 .  .  .  35.15  525 comissões. 3!(7  3)! 4!(6  4)! 3!.4! 4!.2! 3! 2 Prof: Linhares e Júlio Página 5
  • 6. Binômio de Newton Introdução Pelos produtos notáveis, sabemos que (a+b)² = a² + 2ab + b². Se quisermos calcular (a + b)³, podemos escrever: (a + b)3 = a3 + 3a2b + 3ab2 + b3 Se quisermos calcular , podemos adotar o mesmo procedimento: (a + b)4 = (a + b)3 (a+b) = (a3 + 3a2b + 3ab2 + b3) (a+b) = a4 + 4a3b + 6a2b2 + 4ab3 + b4 De modo análogo, podemos calcular as quintas e sextas potências e, de modo geral, obter o desenvolvimento da potência a partir da anterior, ou seja, de . Porém quando o valor de n é grande, este processo gradativo de cálculo é muito trabalhoso. Existe um método para desenvolver a enésima potência de um binômio, conhecido como binômio de Newton (Isaac Newton, matemático e físico inglês, 1642 - 1727). Para esse método é necessário saber o que são coeficientes binomiais, algumas de suas propriedades e o triângulo de Pascal. Coeficientes Binomiais Sendo n e p dois números naturais , chamamos de coeficiente binomial de classe p, do número n, o número , que indicamos por (lê-se: n sobre p). Podemos escrever: O coeficiente binomial também é chamado de número binomial. Por analogia com as frações, dizemos que n é o seu numerador e p, o denominador. Podemos escrever: Prof: Linhares e Júlio Página 6
  • 7. É também imediato que, para qualquer n natural, temos: Exemplos: Propriedades dos coeficientes binomiais Se n, p, k e p + k = n 1ª) então Coeficientes binomiais como esses, que tem o mesmo numerador e a soma dos denominadores igual ao numerador, são chamados complementares. Exemplos: 2ª) Se n, p, k e p p-1 0 Prof: Linhares e Júlio Página 7
  • 8. então Essa igualdade é conhecida como relação de Stifel (Michael Stifel, matemático alemão, 1487 - 1567). Exemplos: Triângulo de Pascal A disposição ordenada dos números binomiais, como na tabela ao lado, recebe o nome de Triângulo de Pascal Nesta tabela triangular, os números binomiais com o mesmo numerador são escritos na mesma linha e os de mesmo denominador, na mesma coluna. Por exemplo, os números binomiais , , e estão na linha 3 e os números binomiais , , , , ..., , ... estão na coluna 1. Substituindo cada número binomial pelo seu respectivo valor, temos: Prof: Linhares e Júlio Página 8
  • 9. Construção do triângulo de Pascal Para construir o triângulo do Pascal, basta lembrar as seguintes propriedades dos números binomiais, não sendo necessário calculá-los: 1ª) Como = 1, todos os elementos da coluna 0 são iguais a 1. 2ª) Como = 1, o último elemento de cada linha é igual a 1. 3ª) Cada elemento do triângulo que não seja da coluna 0 nem o último de cada linha é igual à soma daquele que está na mesma coluna e linha anterior com o elemento que se situa à esquerda deste último (relação de Stifel). Observe os passos e aplicação da relação de Stifel para a construção do triângulo: Prof: Linhares e Júlio Página 9
  • 10. Propriedade do triângulo de Pascal P1 Em Qualquer linha, dois números binomiais eqüidistantes dos extremos são iguais. De fato, esses binomiais são complementares. P2 Teorema das linhas: A soma dos elementos da enésima linha é . De modo geral temos: P3 Teorema das colunas: A soma dos elementos de qualquer coluna, do 1º elemento até um qualquer, é igual ao elemento situado na coluna à direita da considerada e na linha imediatamente abaixo. Prof: Linhares e Júlio Página 10
  • 11. 1 + 2 + 3 + 4 + 5 + 6 = 21 1 + 4 + 10 + 20 = 35 P4 Teorema das diagonais: A soma dos elementos situados na mesma diagonal desde o elemento da 1ª coluna até o de uma qualquer é igual ao elemento imediatamente abaixo deste. 1 + 3 + 6 + 10 + 15 = 35 Fórmula do desenvolvimento do binômio de Newton Como vimos, a potência da forma , em que a, ,é chamada binômio de Newton. Além disso: quando n = 0 temos quando n = 1 temos quando n = 2 temos quando n = 3 temos quando n = 4 temos Prof: Linhares e Júlio Página 11
  • 12. Observe que os coeficientes dos desenvolvimentos foram o triângulo de Pascal. Então, podemos escrever também: De modo geral, quando o expoente é n, podemos escrever a fórmula do desenvolvimento do binômio de Newton: Note que os expoentes de a vão diminuindo de unidade em unidade, variando de n até 0, e os expoentes de b vão aumentando de unidade em unidade, variando de 0 até n. O desenvolvimento de (a + b)n possui n + 1 termos. Prof: Linhares e Júlio Página 12
  • 13. Fórmula do termo geral do binômio Observando os termos do desenvolvimento de (a + b)n, notamos que cada um deles é da forma . Quando p = 0 temos o 1º termo: Quando p = 1 temos o 2º termo: Quando p = 2 temos o 3º termo: Quando p = 3 temos o 4º termo: Quando p = 4 temos o 5º termo: .............................................................................. Percebemos, então, que um termo qualquer T de ordem p + 1pode ser expresso por: Prof: Linhares e Júlio Página 13
  • 14. Cilindro Na figura abaixo, temos dois planos paralelos e distintos, , um círculo R contido em e uma reta r que intercepta , mas não R: Para cada ponto C da região R, vamos considerar o segmento , paralelo à reta r : Assim, temos: Prof: Linhares e Júlio Página 14
  • 15. Chamamos de cilindro, ou cilindro circular, o conjunto de todos os segmentos congruentes e paralelos a r. Elementos do cilindro Dado o cilindro a seguir, consideramos os seguintes elementos: bases: os círculos de centro O e O'e raios r altura: a distância h entre os planos geratriz: qualquer segmento de extremidades nos pontos das circunferências das bases ( por exemplo, ) e paralelo à reta r . Prof: Linhares e Júlio Página 15
  • 16. Áreas Num cilindro, consideramos as seguintes áreas: a) área lateral (AL) Podemos observar a área lateral de um cilindro fazendo a sua planificação: Assim, a área lateral do cilindro reto cuja altura é h e cujos raios dos círculos das bases são r é um retângulo de dimensões : b) área da base ( AB):área do círculo de raio r c) área total ( AT): soma da área lateral com as áreas das bases Prof: Linhares e Júlio Página 16
  • 17. Volume Para obter o volume do cilindro, vamos usar novamente o princípio de Cavalieri. Dados dois sólidos com mesma altura e um plano , se todo plano , paralelo ao plano , intercepta os sólidos e determina secções de mesma área, os sólidos têm volumes iguais: Se 1 é um paralelepípedo retângulo, então V2 = ABh. Assim, o volume de todo paralelepípedo retângulo e de todo cilindro é o produto da área da base pela medida de sua altura: Vcilindro = ABh No caso do cilindro circular reto, a área da base é a área do círculo de raio r ; portanto seu volume é: Prof: Linhares e Júlio Página 17
  • 18. Esfera Chamamos de esfera de centro O e raio R o conjunto de pontos do espaço cuja distância ao centro é menor ou igual ao raio R. Considerando a rotação completa de um semicírculo em torno de um eixo e, a esfera é o sólido gerado por essa rotação. Assim, ela é limitada por uma superfície esférica e formada por todos os pontos pertencentes a essa superfície e ao seu interior. Volume O volume da esfera de raio R é dado por: Prof: Linhares e Júlio Página 18
  • 19. Partes da esfera Superfície esférica A superfície esférica de centro O e raio R é o conjunto de pontos do es[aço cuja distância ao ponto O é igual ao raio R. Se considerarmos a rotação completa de uma semicircunferência em torno de seu diâmetro, a superfície esférica é o resultado dessa rotação. A área da superfície esférica é dada por: Cone circular Dado um círculo C, contido num plano , e um ponto V ( vértice) fora de , chamamos de cone circular o conjunto de todos os segmentos . Prof: Linhares e Júlio Página 19
  • 20. Elementos do cone circular Dado o cone a seguir, consideramos os seguintes elementos: altura: distância h do vértice V ao plano geratriz (g):segmento com uma extremidade no ponto V e outra num ponto da circunferência raio da base: raio R do círculo eixo de rotação:reta determinada pelo centro do círculo e pelo vértice do cone Cone reto Todo cone cujo eixo de rotação é perpendicular à base é chamado cone reto, também denominado cone de revolução. Ele pode ser gerado pela rotação completa de um triângulo retângulo em torno de um de seus catetos. Prof: Linhares e Júlio Página 20
  • 21. Da figura, e pelo Teorema de Pitágoras, temos a seguinte relação: G2 = h2 + R2 Secção meridiana A secção determinada, num cone de revolução, por um plano que contém o eixo de rotação é chamada secção meridiana. Se o triângulo AVB for eqüilátero, o cone também será eqüilátero: Prof: Linhares e Júlio Página 21
  • 22. Áreas Desenvolvendo a superfície lateral de um cone circular reto, obtemos um setor circular de raio g e comprimento : Assim, temos de considerar as seguintes áreas: a) área lateral (AL): área do setor circular b) área da base (AB):área do circulo do raio R c) área total (AT):soma da área lateral com a área da base Prof: Linhares e Júlio Página 22
  • 23. Volume Para determinar o volume do cone, vamos ver como calcular volumes de sólidos de revolução. Observe a figura: d = distância do centro de gravidade (CG) da sua superfície ao eixo e S=área da superfície Sabemos, pelo Teorema de Pappus - Guldin, que, quando uma superfície gira em torno de um eixo e, gera um volume tal que: Vamos, então, determinar o volume do cone de revolução gerado pela rotação de um triângulo retângulo em torno do cateto h: O CG do triângulo está a uma distância do eixo de rotação. Logo: Prof: Linhares e Júlio Página 23
  • 24. CONJUNTOS NUMÉRICOS  Conjunto dos números naturais (IN) IN={0, 1, 2, 3, 4, 5,...} Um subconjunto importante de IN é o conjunto IN*: IN*={1, 2, 3, 4, 5,...}  o zero foi excluído do conjunto IN. Podemos considerar o conjunto dos números naturais ordenados sobre uma reta, como mostra o gráfico abaixo:  Conjunto dos números inteiros (Z) Z={..., -3, -2, -1, 0, 1, 2, 3,...} O conjunto IN é subconjunto de Z. Temos também outros subconjuntos de Z: Z* = Z-{0} Z+ = conjunto dos inteiros não negativos = {0,1,2,3,4,5,...} Z_ = conjunto dos inteiros não positivos = {0,-1,-2,-3,-4,-5,...} Observe que Z+=IN. Podemos considerar os números inteiros ordenados sobre uma reta, conforme mostra o gráfico abaixo: Prof: Linhares e Júlio Página 24
  • 25.  Conjunto dos números racionais (Q) Os números racionais são todos aqueles que podem ser colocados na forma de fração (com o numerador e denominador  Z). Ou seja, o conjunto dos números racionais é a união do conjunto dos números inteiros com as frações positivas e negativas. 5 3 3 Então : -2,  ,  1, , 1, , por exemplo, são números racionais. 4 5 2 Exemplos: 3 6 9 a)  3    1 2 3 1 2 3 b) 1    1 2 3 Assim, podemos escrever: a Q  {x | x  , com a  Z , b  Z e b  0} b É interessante considerar a representação decimal de um número racional a , que se obtém dividindo a por b. b Exemplos referentes às decimais exatas ou finitas: 1 5 75  0,5   1,25  3,75 2 4 20 Exemplos referentes às decimais periódicas ou infinitas: 1 6 7  0,333...  0,857142857142...  1,1666... 3 7 6 Toda decimal exata ou periódica pode ser representada na forma de número racional. Prof: Linhares e Júlio Página 25
  • 26.  Conjunto dos números irracionais Os números irracionais são decimais infinitas não periódicas, ou seja, os números que não podem ser escrito na forma de fração (divisão de dois inteiros). Como exemplo de números irracionais, temos a raiz quadrada de 2 e a raiz quadrada de 3: 2  1,4142135... 3  1,7320508... Um número irracional bastante conhecido é o número =3,1415926535...  Conjunto dos números reais (IR) Dados os conjuntos dos números racionais (Q) e dos irracionais, definimos o conjunto dos números reais como: IR=Q  {irracionais} = {x|x é racional ou x é irracional} O diagrama abaixo mostra a relação entre os conjuntos numéricos: Portanto, os números naturais, inteiros, racionais e irracionais são todos números reais. Como subconjuntos importantes de IR temos: IR* = IR-{0} IR+ = conjunto dos números reais não negativos IR_ = conjunto dos números reais não positivos Obs: entre dois números inteiros existem infinitos números reais. Por exemplo:  Entre os números 1 e 2 existem infinitos números reais: 1,01 ; 1,001 ; 1,0001 ; 1,1 ; 1,2 ; 1,5 ; 1,99 ; 1,999 ; 1,9999 ...  Entre os números 5 e 6 existem infinitos números reais: 5,01 ; 5,02 ; 5,05 ; 5,1 ; 5,2 ; 5,5 ; 5,99 ; 5,999 ; 5,9999 ... Prof: Linhares e Júlio Página 26
  • 27. Determinantes Como já vimos, matriz quadrada é a que tem o mesmo número de linhas e de colunas (ou seja, é do tipo nxn). A toda matriz quadrada está associado um número ao qual damos o nome de determinante. Dentre as várias aplicações dos determinantes na Matemática, temos: resolução de alguns tipos de sistemas de equações lineares; cálculo da área de um triângulo situado no plano cartesiano, quando são conhecidas as coordenadas dos seus vértices; Determinante de 1ª ordem Dada uma matriz quadrada de 1ª ordem M=[a11], o seu determinante é o número real a11: det M =Ia11I = a11 Observação: Representamos o determinante de uma matriz entre duas barras verticais, que não têm o significado de módulo. Por exemplo: M= [5] det M = 5 ou I 5 I = M = [-3] det M = -3 ou I -3 5 I = -3 Determinante de 2ª ordem Dada a matriz , de ordem 2, por definição o determinante associado a M, determinante de 2ª ordem, é dado por: Prof: Linhares e Júlio Página 27
  • 28. Portanto, o determinante de uma matriz de ordem 2 é dado pela diferença entre o produto dos elementos da diagonal principal e o produto dos elementos da diagonal secundária. Veja o exemplo a seguir. Menor complementar Chamamos de menor complementar relativo a um elemento aij de uma matriz M, quadrada e de ordem n>1, o determinante MCij , de ordem n - 1, associado à matriz obtida de M quando suprimimos a linha e a coluna que passam por aij . Vejamos como determiná-lo pelos exemplos a seguir: a) Dada a matriz , de ordem 2, para determinar o menor complementar relativo ao elemento a11(MC11), retiramos a linha 1 e a coluna 1: Da mesma forma, o menor complementar relativo ao elemento a12 é: Prof: Linhares e Júlio Página 28
  • 29. b) Sendo , de ordem 3, temos: Cofator Chamamos de cofator ou complemento algébrico relativo a um elemento aij de uma matriz quadrada de ordem n o número Aij tal que Aij = (-1)i+j . MCij . Veja: a) Dada , os cofatores relativos aos elementos a11 e a12 da matriz M são: b) Sendo , vamos calcular os cofatores A22, A23 e A31: Prof: Linhares e Júlio Página 29
  • 30. Teorema de Laplace O determinante de uma matriz quadrada M = [aij]mxn pode ser obtido pela soma dos produtos dos elementos de uma fila qualquer ( linha ou coluna) da matriz M pelos respectivos cofatores. Assim, fixando , temos: em que é o somatório de todos os termos de índice i, variando de 1 até m, . Regra de Sarrus O cálculo do determinante de 3ª ordem pode ser feito por meio de um dispositivo prático, denominado regra de Sarrus. Acompanhe como aplicamos essa regra para . 1º passo: Repetimos as duas primeiras colunas ao lado da terceira: Prof: Linhares e Júlio Página 30
  • 31. 2º passo: Encontramos a soma do produto dos elementos da diagonal principal com os dois produtos obtidos pela multiplicação dos elementos das paralelas a essa diagonal (a soma deve ser precedida do sinal positivo): 3º passo: Encontramos a soma do produto dos elementos da diagonal secundária com os dois produtos obtidos pela multiplicação dos elementos das paralelas a essa diagonal ( a soma deve ser precedida do sinal negativo): Assim: Prof: Linhares e Júlio Página 31
  • 32. Observação: Se desenvolvermos esse determinante de 3ª ordem aplicando o Teorema de Laplace, encontraremos o mesmo número real. Determinante de ordem n > 3 Vimos que a regra de Sarrus é válida para o cálculo do determinante de uma matriz de ordem 3. Quando a matriz é de ordem superior a 3, devemos empregar o Teorema de Laplace para chegar a determinantes de ordem 3 e depois aplicar a regra de Sarrus. Propriedades dos determinantes Os demais associados a matrizes quadradas de ordem n apresentam as seguintes propriedades: P1 ) Quando todos os elementos de uma fila ( linha ou coluna) são nulos, o determinante dessa matriz é nulo. Exemplo: P2) Se duas filas de uma matriz são iguais, então seu determinante é nulo. Exemplo: P3) Se duas filas paralelas de uma matriz são proporcionais, então seu determinante é nulo. Exemplo: Prof: Linhares e Júlio Página 32
  • 33. P4) Se os elementos de uma fila de uma matriz são combinações lineares dos elementos correspondentes de filas paralelas, então seu determinante é nulo. Exemplos: P5 ) Teorema de Jacobi: o determinante de uma matriz não se altera quando somamos aos elementos de uma fila uma combinação linear dos elementos correspondentes de filas paralelas. Exemplo: Substituindo a 1ª coluna pela soma dessa mesma coluna com o dobro da 2ª, temos: P6) O determinante de uma matriz e o de sua transposta são iguais. Exemplo: Prof: Linhares e Júlio Página 33
  • 34. P7) Multiplicando por um número real todos os elementos de uma fila em uma matriz, o determinante dessa matriz fica multiplicado por esse número. Exemplos: P8) Quando trocamos as posições de duas filas paralelas, o determinante de uma matriz muda de sinal. Exemplo: P9) Quando, em uma matriz, os elementos acima ou abaixo da diagonal principal são todos nulos, o determinante é igual ao produto dos elementos dessa diagonal. Exemplos: Prof: Linhares e Júlio Página 34
  • 35. P10) Quando, em uma matriz, os elementos acima ou abaixo da diagonal secundária são todos nulos, o determinante é igual ao produto dos elementos dessa diagonal multiplicado por . Exemplos: P11) Para A e B matrizes quadradas de mesma ordem n, . Como: Exemplo: P12) Exemplo: Prof: Linhares e Júlio Página 35
  • 36. Equações algébricas (com uma variável) Introdução Equação é toda sentença matemática aberta que exprime uma relação de igualdade. A palavra equação tem o prefixo equa, que em latim quer dizer "igual". Exemplos: 2x + 8 = 0 5x - 4 = 6x + 8 3a - b - c = 0 Não são equações: 4 + 8 = 7 + 5 (Não é uma sentença aberta) x - 5 < 3 (Não é igualdade) (não é sentença aberta, nem igualdade) A equação geral do primeiro grau: ax+b = 0 onde a e b são números conhecidos e a > 0, se resolve de maneira simples: subtraindo b dos dois lados, obtemos: ax = -b dividindo agora por a (dos dois lados), temos: Considera a equação 2x - 8 = 3x -10 Prof: Linhares e Júlio Página 36
  • 37. A letra é a incógnita da equação. A palavra incógnita significa " desconhecida". Na equação acima a incógnita é x; tudo que antecede o sinal da igualdade denomina-se 1º membro, e o que sucede, 2º membro. Qualquer parcela, do 1º ou do 2º membro, é um termo da equação. Equação do 1º grau na incógnita x é toda equação que pode ser escrita na forma ax=b, sendo a e b números racionais, com a diferente de zero. Conjunto Verdade e Conjunto Universo de uma Equação Considere o conjunto A = {0, 1, 2, 3, 4, 5} e a equação x + 2 = 5. Observe que o número 3 do conjunto A é denominado conjunto universo da equação e o conjunto {3} é o conjunto verdade dessa mesma equação. Observe este outro exemplo: Determine os números inteiros que satisfazem a equação x² = 25 O conjunto dos números inteiro é o conjunto universo da equação. Prof: Linhares e Júlio Página 37
  • 38. Os números -5 e 5, que satisfazem a equação, formam o conjunto verdade, podendo ser indicado por: V = {-5, 5}. Daí concluímos que: Conjunto Universo é o conjunto de todos os valores que variável pode assumir. Indica-se por U. Conjunto verdade é o conjunto dos valores de U, que tornam verdadeira a equação . Indica-se por V. Observações: O conjunto verdade é subconjunto do conjunto universo. Não sendo citado o conjunto universo, devemos considerar como conjunto universo o conjunto dos números racionais. O conjunto verdade é também conhecido por conjunto solução e pode ser indicado por S. Raízes de uma equação Os elementos do conjunto verdade de uma equação são chamados raízes da equação. Para verificar se um número é raiz de uma equação, devemos obedecer à seguinte seqüência: Substituir a incógnita por esse número. Determinar o valor de cada membro da equação. Verificar a igualdade, sendo uma sentença verdadeira, o número considerado é raiz da equação. Prof: Linhares e Júlio Página 38
  • 39. Exemplos: Verifique quais dos elementos do conjunto universo são raízes das equações abaixo, determinando em cada caso o conjunto verdade. Resolva a equação x - 2 = 0, sendo U = {0, 1, 2, 3}. Para x = 0 na equação x - 2 = 0 temos: 0 - 2 = 0 => -2 = 0. (F) Para x = 1 na equação x - 2 = 0 temos: 1 - 2 = 0 => -1 = 0. (F) Para x = 2 na equação x - 2 = 0 temos: 2 - 2 = 0 => 0 = 0. (V) Para x = 3 na equação x - 2 = 0 temos: 3 - 2 = 0 => 1 = 0. (F) Verificamos que 2 é raiz da equação x - 2 = 0, logo V = {2}. Resolva a equação 2x - 5 = 1, sendo U = {-1, 0, 1, 2}. Para x = -1 na equação 2x - 5 = 1 temos: 2 . (-1) - 5 = 1 => -7 = 1. (F) Para x = 0 na equação 2x - 5 = 1 temos: 2 . 0 - 5 = 1 => -5 = 1. (F) Para x = 1 na equação 2x - 5 = 1 temos: 2 . 1 - 5 = 1 => -3 = 1. (F) Para x = 2 na equação 2x - 5 = 1 temos: 2 . 2 - 5 = 1 => -1 = 1. (F) A equação 2x - 5 = 1 não possui raiz em U, logo V = Ø. Prof: Linhares e Júlio Página 39
  • 40. Função de 1º grau - Afim Definição Chama-se função polinomial do 1º grau, ou função afim, a qualquer função f de IR em IR dada por uma lei da forma f(x) = ax + b, onde a e b são números reais dados e a 0. Na função f(x) = ax + b, o número a é chamado de coeficiente de x e o número b é chamado termo constante. Veja alguns exemplos de funções polinomiais do 1º grau: f(x) = 5x - 3, onde a = 5 e b = - 3 f(x) = -2x - 7, onde a = -2 e b = - 7 f(x) = 11x, onde a = 11 e b = 0 Gráfico O gráfico de uma função polinomial do 1º grau, y = ax + b, com a 0, é uma reta oblíqua aos eixos Ox e Oy. Exemplo: Vamos construir o gráfico da função y = 3x - 1: Como o gráfico é uma reta, basta obter dois de seus pontos e ligá-los com o auxílio de uma régua: a) Para x = 0, temos y = 3 · 0 - 1 = -1; portanto, um ponto é (0, -1). b) Para y = 0, temos 0 = 3x - 1; portanto, e outro ponto é . Marcamos os pontos (0, -1) e no plano cartesiano e ligamos os dois com uma reta. Prof: Linhares e Júlio Página 40
  • 41. x y 0 -1 0 Já vimos que o gráfico da função afim y = ax + b é uma reta. O coeficiente de x, a, é chamado coeficiente angular da reta e, como veremos adiante, a está ligado à inclinação da reta em relação ao eixo Ox. O termo constante, b, é chamado coeficiente linear da reta. Para x = 0, temos y = a · 0 + b = b. Assim, o coeficiente linear é a ordenada do ponto em que a reta corta o eixo Oy. Zero e Equação do 1º Grau Chama-se zero ou raiz da função polinomial do 1º grau f(x) = ax + b, a 0, o número real x tal que f(x) = 0. Temos: f(x) = 0 ax + b = 0 Vejamos alguns exemplos: 1. Obtenção do zero da função f(x) = 2x - 5: f(x) = 0 2x - 5 = 0 2. Cálculo da raiz da função g(x) = 3x + 6: g(x) = 0 3x + 6 = 0 x = -2 3. Cálculo da abscissa do ponto em que o gráfico de h(x) = -2x + 10 corta o eixo das abicissas: O ponto em que o gráfico corta o eixo dos x é aquele em que h(x) = Prof: Linhares e Júlio Página 41
  • 42. 0; então: h(x) = 0 -2x + 10 = 0 x=5 Crescimento e decrescimento Consideremos a função do 1º grau y = 3x - 1. Vamos atribuir valores cada vez maiores a x e observar o que ocorre com y: x -3 -2 -1 0 1 2 3 y -10 -7 -4 -1 2 5 8 Notemos que, quando aumentos o valor de x, os correspondentes valores de y também aumentam. Dizemos, então que a função y = 3x - 1 é crescente. Observamos novamente seu gráfico: Regra geral: a função do 1º grau f(x) = ax + b é crescente quando o coeficiente de x é positivo (a > 0); a função do 1º grau f(x) = ax + b é decrescente quando o coeficiente de x é negativo (a < 0); Prof: Linhares e Júlio Página 42
  • 43. Justificativa: para a > 0: se x1 < x2, então ax1 < ax2. Daí, ax1 + b < ax2 + b, de onde vem f(x1) < f(x2). para a < 0: se x1 < x2, então ax1 > ax2. Daí, ax1 + b > ax2 + b, de onde vem f(x1) > f(x2). Sinal Estudar o sinal de uma qualquer y = f(x) é determinar os valor de x para os quais y é positivo, os valores de x para os quais y é zero e os valores de x para os quais y é negativo. Consideremos uma função afim y = f(x) = ax + b vamos estudar seu sinal. Já vimos que essa função se anula pra raiz . Há dois casos possíveis: 1º) a > 0 (a função é crescente) y>0 ax + b > 0 x> y>0 ax + b < 0 x< Conclusão: y é positivo para valores de x maiores que a raiz; y é negativo para valores de x menores que a raiz 2º) a < 0 (a função é decrescente) y>0 ax + b > 0 x< Prof: Linhares e Júlio Página 43
  • 44. y>0 ax + b < 0 x< Conclusão: y é positivo para valores de x menores que a raiz; y é negativo para valores de x maiores que a raiz. Prof: Linhares e Júlio Página 44
  • 45. EQUAÇÕES EXPONENCIAIS Chamamos de equações exponenciais toda equação na qual a incógnita aparece em expoente. Exemplos de equações exponenciais: 1) 3x =81 (a solução é x=4) 2) 2x-5=16 (a solução é x=9) 3) 16x-42x-1-10=22x-1 (a solução é x=1) 4) 32x-1-3x-3x-1+1=0 (as soluções são x’=0 e x’’=1) Para resolver equações exponenciais, devemos realizar dois passos importantes: 1º) redução dos dois membros da equação a potências de mesma base; 2º) aplicação da propriedade: a m  a n  m  n (a  1 e a  0) EXERCÍCIOS RESOLVIDOS: 1) 3x=81 Resolução: Como 81=34, podemos escrever 3x = 34 E daí, x=4. 2) 9x = 1 Resolução: 9x = 1  9x = 90 ; logo x=0. x 3 81 3)    4 256 x x x 4 3 81 3 34 3 3 Resolução :        4       ; então x  4. 4 256 4 4 4 4 4) 3 x  4 27 3 3 Resolução : 3  27  3  3  3  3 ; logo x  x 4 x 4 3 x 4 4 5) 23x-1 = 322x Prof: Linhares e Júlio Página 45
  • 46. Resolução: 23x-1 = 322x  23x-1 = (25)2x  23x-1 = 210x ; daí 3x-1=10, de onde x=-1/7. 6) Resolva a equação 32x–6.3x–27=0. Resolução: vamos resolver esta equação através de uma transformação: 32x–6.3x–27=0  (3x)2-6.3x–27=0 Fazendo 3x=y, obtemos: y2-6y–27=0 ; aplicando Bhaskara encontramos  y’=-3 e y’’=9 Para achar o x, devemos voltar os valores para a equação auxiliar 3x=y: y’=-3  3x’ = -3  não existe x’, pois potência de base positiva é positiva y’’=9  3x’’ = 9  3x’’ = 32  x’’=2 Portanto a solução é x=2 FUNÇÃO EXPONENCIAL Chamamos de funções exponenciais aquelas nas quais temos a variável aparecendo em expoente. A função f:IRIR+ definida por f(x)=ax, com a  IR+ e a1, é chamada função exponencial de base a. O domínio dessa função é o conjunto IR (reais) e o contradomínio é IR+ (reais positivos, maiores que zero). GRÁFICO CARTESIANO DA FUNÇÃO EXPONENCIAL Temos 2 casos a considerar:  quando a>1;  quando 0<a<1. Acompanhe os exemplos seguintes: 1) y=2x (nesse caso, a=2, logo a>1) Atribuindo alguns valores a x e calculando os correspondentes valores de y, obtemos a tabela e o gráfico abaixo: Prof: Linhares e Júlio Página 46
  • 47. X -2 -1 0 1 2 y 1/4 1/2 1 2 4 2) y=(1/2)x (nesse caso, a=1/2, logo 0<a<1) Atribuindo alguns valores a x e calculando os correspondentes valores de y, obtemos a tabela e o gráfico abaixo: X -2 -1 0 1 2 Y 4 2 1 1/2 1/4 Nos dois exemplos, podemos observar que a) o gráfico nunca intercepta o eixo horizontal; a função não tem raízes; b) o gráfico corta o eixo vertical no ponto (0,1); c) os valores de y são sempre positivos (potência de base positiva é positiva), portanto o conjunto imagem é Im=IR+. Além disso, podemos estabelecer o seguinte: Prof: Linhares e Júlio Página 47
  • 48. a>1 0<a<1 f(x) é crescente e Im=IR+ f(x) é decrescente e Im=IR+ Para quaisquer x1 e x2 do domínio: Para quaisquer x1 e x2 do domínio: x2>x1  y2>y1 (as desigualdades têm x2>x1  y2<y1 (as desigualdades têm mesmo sentido) sentidos diferentes) INEQUAÇÕES EXPONENCIAIS Chamamos de inequações exponenciais toda inequação na qual a incógnita aparece em expoente. Exemplos de inequações exponenciais: 1) 3 x  81 (a solução é x  4) 1 2) 2 2x-2  2 x 2 (que é satisfeita para todo x real) x 3 4 4 3)      (que é satisfeita para x  -3) 5 5 4) 25 x - 150.5 x  3125  0 (que é satisfeita para 2  x  3) Para resolver inequações exponenciais, devemos realizar dois passos importantes: 1º) redução dos dois membros da inequação a potências de mesma base; 2º) aplicação da propriedade: a>1 0<a<1 am > an  m>n am > an  m<n (as desigualdades têm mesmo sentido) (as desigualdades têm sentidos diferentes) Prof: Linhares e Júlio Página 48
  • 49. EXERCÍCIO RESOLVIDO:  11 1) 4 x 1  4 x  4 x 1  4 Resolução : 4x  11 A inequação pode ser escrita  4 x  4 x .4  . 4 4 M ultiplicando ambos os lados por 4 temos : 4 x  4.4 x  16.4 x  11 , ou seja : (1  4  16).4 x  11  -11.4 x  11 e daí, 4 x  1 Porém, 4 x  1  4 x  4 0. Como a base (4) é maior que 1, obtemos : 4 x  40  x  0 Portanto S  IR - (reais negativos) Prof: Linhares e Júlio Página 49
  • 50. FUNÇÃO LOGARÍTMICA A função f:IR+IR definida por f(x)=logax, com a1 e a>0, é chamada função logarítmica de base a. O domínio dessa função é o conjunto IR+ (reais positivos, maiores que zero) e o contradomínio é IR (reais). GRÁFICO CARTESIANO DA FUNÇÃO LOGARÍTMICA Temos 2 casos a considerar:  quando a>1;  quando 0<a<1. Acompanhe nos exemplos seguintes, a construção do gráfico em cada caso: 3) y=log2x (nesse caso, a=2, logo a>1) Atribuindo alguns valores a x e calculando os correspondentes valores de y, obtemos a tabela e o gráfico abaixo: x 1/4 1/2 1 2 4 y -2 -1 0 1 2 4) y=log(1/2)x (nesse caso, a=1/2, logo 0<a<1) Atribuindo alguns valores a x e calculando os correspondentes valores de y, obtemos a tabela e o gráfico abaixo: Prof: Linhares e Júlio Página 50
  • 51. x 1/4 1/2 1 2 4 y 2 1 0 -1 -2 Nos dois exemplos, podemos observar que d) o gráfico nunca intercepta o eixo vertical; e) o gráfico corta o eixo horizontal no ponto (1,0). A raiz da função é x=1; f) y assume todos os valores reais, portanto o conjunto imagem é Im=IR. Além disso, podemos estabelecer o seguinte: a>1 0<a<1 f(x) é crescente e Im=IR f(x) é decrescente e Im=IR Para quaisquer x1 e x2 do domínio: Para quaisquer x1 e x2 do domínio: x2>x1  y2>y1 (as desigualdades têm x2>x1  y2<y1 (as desigualdades têm mesmo sentido) sentidos diferentes) Prof: Linhares e Júlio Página 51
  • 52. EQUAÇÕES LOGARÍTMICAS Chamamos de equações logarítmicas toda equação que envolve logaritmos com a incógnita aparecendo no logaritmando, na base ou em ambos. Exemplos de equações logarítmicas: 7) log3x =5 (a solução é x=243) 8) log(x2-1) = log 3 (as soluções são x’=-2 e x’’=2) 9) log2(x+3) + log2(x-3) = log27 (a solução é x=4) 10) logx+1(x2-x)=2 (a solução é x=-1/3) Alguns exemplos resolvidos: 1) log3(x+5) = 2 Resolução: condição de existência: x+5>0 => x>-5 log3(x+5) = 2 => x+5 = 32 => x=9-5 => x=4 Como x=4 satisfaz a condição de existência, então o conjunto solução é S={4}. 2) log2(log4 x) = 1 Resolução: condição de existência: x>0 e log4x>0 log2(log4 x) = 1 ; sabemos que 1 = log2(2), então log2(log4x) = log2(2) => log4x = 2 => 42 = x => x=16 Como x=16 satisfaz as condições de existência, então o conjunto solução é S={16}. 3) Resolva o sistema: log x  log y  7  3. log x  2. log y  1 Resolução: condições de existência: x>0 e y>0 Da primeira equação temos: log x+log y=7 => log y = 7-log x Substituindo log y na segunda equação temos: 3.log x – 2.(7-log x)=1 => 3.log x-14+2.log x = 1 => 5.log x = 15 => => log x =3 => x=103 Substituindo x= 103 em log y = 7-log x temos: Prof: Linhares e Júlio Página 52
  • 53. log y = 7- log 103 => log y = 7-3 => log y =4 => y=104. Como essas raízes satisfazem as condições de existência, então o conjunto solução é S={(103;104)}. INEQUAÇÕES LOGARÍTMICAS Chamamos de inequações logarítmicas toda inequação que envolve logaritmos com a incógnita aparecendo no logaritmando, na base ou em ambos. Exemplos de inequações logarítmicas: 1) log2x > 0 (a solução é x>1) 2) log4(x+3)  1 (a solução é –3<x1) Para resolver inequações logarítmicas, devemos realizar dois passos importantes: 1º) redução dos dois membros da inequação a logaritmos de mesma base; 2º) aplicação da propriedade: a>1 0<a<1 logam > logan  m>n>0 logam > logan  0<m<n (as desigualdades têm mesmo sentido) (as desigualdades têm sentidos diferentes) EXERCÍCIOS RESOLVIDOS: 1) log2(x+2) > log28 Resolução: Condições de existência: x+2>0, ou seja, x>-2 (S1) Como a base (2) é maior que 1, temos: x+2>8 e, daí, x>6 (S2) O conjunto solução é S= S1  S2 = {x  IR| x>6}. Portanto a solução final é a intersecção de S1 e S2, como está representado logo abaixo no desenho: Prof: Linhares e Júlio Página 53
  • 54. 2) log2(log3x)  0 Resolução: Condições de existência: x>0 e log3x>0 Como log21=0, a inequação pode ser escrita assim: log2(log3x)  log21 Sendo a base (2) maior que 1, temos: log3x  1. Como log33 = 1, então, log3x  log33 e, daí, x  3, porque a base (3) é maior que 1. As condições de existência estão satisfeitas, portanto S={x  IR| x  3}. Prof: Linhares e Júlio Página 54
  • 55. Função Quadrática Definição Chama-se função quadrática, ou função polinomial do 2º grau, qualquer função f de IR em IR dada por uma lei da forma f(x) = ax2 + bx + c, onde a, b e c são números reais e a 0. Vejamos alguns exemplos de função quadráticas: 1. f(x) = 3x2 - 4x + 1, onde a = 3, b = - 4 e c = 1 2. f(x) = x2 -1, onde a = 1, b = 0 e c = -1 3. f(x) = 2x2 + 3x + 5, onde a = 2, b = 3 e c = 5 4. f(x) = - x2 + 8x, onde a = 1, b = 8 e c = 0 5. f(x) = -4x2, onde a = - 4, b = 0 e c = 0 Gráfico O gráfico de uma função polinomial do 2º grau, y = ax2 + bx + c, com a 0, é uma curva chamada parábola. Exemplo: Vamos construir o gráfico da função y = x 2 + x: Primeiro atribuímos a x alguns valores, depois calculamos o valor correspondente de y e, em seguida, ligamos os pontos assim obtidos. x y -3 6 -2 2 -1 0 0 0 1 2 2 6 Prof: Linhares e Júlio Página 55
  • 56. Observação: Ao construir o gráfico de uma função quadrática y = ax2 + bx + c, notaremos sempre que: se a > 0, a parábola tem a concavidade voltada para cima; se a < 0, a parábola tem a concavidade voltada para baixo; Zero e Equação do 2º Grau Chama-se zeros ou raízes da função polinomial do 2º grau f(x) = ax2 + bx + c , a 0, os números reais x tais que f(x) = 0. Então as raízes da função f(x) = ax2 + bx + c são as soluções da equação do 2º grau ax2 + bx + c = 0, as quais são dadas pela chamada fórmula de Bhaskara: Temos: Observação A quantidade de raízes reais de uma função quadrática depende do valor obtido para o radicando , chamado discriminante, a saber: quando é positivo, há duas raízes reais e distintas; quando é zero, há só uma raiz real; quando é negativo, não há raiz real. Coordenadas do vértice da parábola Quando a > 0, a parábola tem concavidade voltada para cima e um ponto de mínimo V; quando a < 0, a parábola tem concavidade voltada para baixo e um ponto de máximo V. Prof: Linhares e Júlio Página 56
  • 57. Em qualquer caso, as coordenadas de V são . Veja os gráficos: Prof: Linhares e Júlio Página 57
  • 58. Imagem O conjunto-imagem Im da função y = ax2 + bx + c, a 0, é o conjunto dos valores que y pode assumir. Há duas possibilidades: 1ª - quando a > 0, a>0 2ª quando a < 0, a<0 Prof: Linhares e Júlio Página 58
  • 59. Construção da Parábola É possível construir o gráfico de uma função do 2º grau sem montar a tabela de pares (x, y), mas seguindo apenas o roteiro de observação seguinte: 1. O valor do coeficiente a define a concavidade da parábola; 2. Os zeros definem os pontos em que a parábola intercepta o eixo dos x; 3. O vértice V indica o ponto de mínimo (se a > 0), ou máximo (se a< 0); 4. A reta que passa por V e é paralela ao eixo dos y é o eixo de simetria da parábola; 5. Para x = 0 , temos y = a · 02 + b · 0 + c = c; então (0, c) é o ponto em que a parábola corta o eixo dos y. Sinal Consideramos uma função quadrática y = f(x) = ax2 + bx + c e determinemos os valores de x para os quais y é negativo e os valores de x para os quais y é positivos. 2 Conforme o sinal do discriminante = b - 4ac, podemos ocorrer os seguintes casos: 1º- >0 Nesse caso a função quadrática admite dois zeros reais distintos (x1 Prof: Linhares e Júlio Página 59
  • 60. x2). a parábola intercepta o eixo Ox em dois pontos e o sinal da função é o indicado nos gráficos abaixo: quando a > 0 y > 0 (x < x1 ou x > x2) y < 0 x1 < x < x2 quando a < 0 y > 0 x1 < x < x2 y<0 (x < x1 ou x > x2) Prof: Linhares e Júlio Página 60
  • 61. 2º - =0 quando a > 0 quando a < 0 Prof: Linhares e Júlio Página 61
  • 62. 3º - <0 quando a > 0 Prof: Linhares e Júlio Página 62
  • 63. quando a < 0 GEOMETRIA ANALÍTICA Retas Introdução Entre os pontos de uma reta e os números reais existe uma correspondência biunívoca, isto é, a cada ponto de reta corresponde um único número real e vice-versa. Considerando uma reta horizontal x, orientada da esquerda para direita (eixo), e determinando um ponto O dessa reta ( origem) e um segmento u, unitário e não-nulo, temos que dois números inteiros e consecutivos determinam sempre nesse eixo um segmento de reta de comprimento u: Medida algébrica de um segmento Fazendo corresponder a dois pontos, A e B, do eixo x os números reais xA e xB , temos: Prof: Linhares e Júlio Página 63
  • 64. A medida algébrica de um segmento orientado é o número real que corresponde à diferença entre as abscissas da extremidade e da origem desse segmento. Plano cartesiano A geometria analítica teve como principal idealizador o filósofo francês René Descartes ( 1596-1650). Com o auxílio de um sistema de eixos associados a um plano, ele faz corresponder a cada ponto do plano um par ordenado e vice-versa. Quando os eixos desse sistemas são perpendiculares na origem, essa correspondência determina um sistema cartesiano ortogonal ( ou plano cartesiano). Assim, há uma reciprocidade entre o estudo da geometria ( ponto, reta, circunferência) e da Álgebra ( relações, equações etc.), podendo-se representar graficamente relações algébricas e expressar algebricamente representações gráficas. Observe o plano cartesiano nos quadros quadrantes: Prof: Linhares e Júlio Página 64
  • 65. Exemplos: A(2, 4) pertence ao 1º quadrante (xA > 0 e yA > 0) B(-3, -5) pertence ao 3º quadrante ( xB < 0 e yB < 0) Observação: Por convenção, os pontos localizados sobre os eixos não estão em nenhum quadrante. Distância entre dois pontos Dados os pontos A(xA, yA) e B(xB, yB) e sendo dAB a distância entre eles, temos: Aplicando o teorema de Pitágoras ao triângulo retângulo ABC, vem: Como exemplo, vamos determinar a distância entre os pontos A(1, -1) e B(4, -5): Prof: Linhares e Júlio Página 65
  • 66. Equações de uma reta Equação geral Podemos estabelecer a equação geral de uma reta a partir da condição de alinhamento de três pontos. Dada uma reta r, sendo A(xA, yA) e B(xB, yB) pontos conhecidos e distintos de r e P(x,y) um ponto genérico, também de r, estando A, B e P alinhados, podemos escrever: Fazendo yA - yB = a, xB - xA = b e xAyB - xByA=c, como a e b não são simultaneamente nulos , temos: ax + by + c = 0 (equação geral da reta r) Essa equação relaciona x e y para qualquer ponto P genérico da reta. Assim, dado o ponto P(m, n): Prof: Linhares e Júlio Página 66
  • 67. se am + bn + c = 0, P é o ponto da reta; se am + bn + c 0, P não é ponto da reta. Acompanhe os exemplos: Vamos considerar a equação geral da reta r que passa por A(1, 3) e B(2, 4). Considerando um ponto P(x, y) da reta, temos: Vamos verificar se os pontos P(-3, -1) e Q(1, 2) pertencem à reta r do exemplo anterior. Substituindo as coordenadas de P em x - y + 2 = 0, temos: -3 - (-1) + 2 = 0 -3 + 1 + 2 = 0 Como a igualdade é verdadeira, então P r. Substituindo as coordenadas de Q em x - y + 2 = 0, obtemos: 1-2+2 0 Como a igualdade não é verdadeira, então Q r. Prof: Linhares e Júlio Página 67
  • 68. Geometria Analítica: Circunferência Equações da circunferência Equação reduzida Circunferência é o conjunto de todos os pontos de um plano eqüidistantes de um ponto fixo, desse mesmo plano, denominado centro da circunferência: Assim, sendo C(a, b) o centro e P(x, y) um ponto qualquer da circunferência, a distância de C a P(dCP) é o raio dessa circunferência. Então: Portanto, (x - a)2 + (y - b)2 =r2 é a equação reduzida da circunferência e permite determinar os elementos essenciais para a construção da circunferência: as coordenadas do centro e o raio. Prof: Linhares e Júlio Página 68
  • 69. Observação: Quando o centro da circunfer6encia estiver na origem ( C(0,0)), a equação da circunferência será x2 + y2 = r2 . Equação geral Desenvolvendo a equação reduzida, obtemos a equação geral da circunferência: Como exemplo, vamos determinar a equação geral da circunferência de centro C(2, -3) e raio r = 4. A equação reduzida da circunferência é: ( x - 2 )2 +( y + 3 )2 = 16 Desenvolvendo os quadrados dos binômios, temos: Geometria Analítica - Cônicas Elipse Considerando, num plano , dois pontos distintos, F1 e F2 , e sendo 2a um número real maior que a distância entre F1 e F2, chamamos de elipse o conjunto dos pontos do plano tais que a soma das distâncias desses pontos a F1 e F2 seja sempre igual a 2a. Por exemplo, sendo P, Q, R, S, F1 e F2 pontos de um mesmo plano e F1F2 < 2a, temos: Prof: Linhares e Júlio Página 69
  • 70. A figura obtida é uma elipse. Observações: 1ª) A Terra descreve uma trajetória elíptica em torno do sol, que é um dos focos dessa trajetória. A lua em torno da terra e os demais satélites em relação a seus respectivos planetas também apresentam esse comportamento. 2ª) O cometa de Halley segue uma órbita elíptica, tendo o Sol como um dos focos. 3ª) As elipses são chamadas cônicas porque ficam configuradas pelo corte feito em um cone circular reto por um plano oblíquo em relação à sua base. Elementos Observe a elipse a seguir. Nela, consideramos os seguintes elementos: Prof: Linhares e Júlio Página 70
  • 71. focos : os pontos F1 e F2 centro: o ponto O, que é o ponto médio de semi-eixo maior: a semi-eixo menor: b semidistância focal: c vértices: os pontos A1, A2, B1, B2 eixo maior: eixo menor: distância focal: Relação fundamental Na figura acima, aplicando o Teorema de Pitágoras ao tri6angulo OF2B2 , retângulo em O, podemos escrever a seguinte relação fundamental: a2 =b2 + c2 Excentricidade Chamamos de excentricidade o número real e tal que: Pela definição de elipse, 2c < 2a, então c < a e, conseqüentemente, 0 < e < 1. Observação:Quando os focos são muito próximos, ou seja, c é muito pequeno, a elipse se aproxima de uma circunferência. Prof: Linhares e Júlio Página 71
  • 72. Equações Vamos considerar os seguintes casos: a) elipse com centro na origem e eixo maior horizontal Sendo c a semidistância focal, os focos da elipse são F1(-c, 0) e F2(c, 0): Aplicando a definição de elipse , obtemos a equação da elipse: b) elipse com centro na origem e eixo maior vertical Nessas condições, a equação da elipse é: Prof: Linhares e Júlio Página 72
  • 73. Hipérbole Considerando, num plano , dois pontos distintos, F1 e F2 , e sendo 2a um número real menor que a distância entre F1 e F2 , chamamos de hipérbole o conjunto dos pontos do plano tais que o módulo da diferença das dist6ancias desses pontos a F1 e F2 seja sempre igual a 2a. Por exemplo, sendo P, Q, R, S, F1 e F2 pontos de um mesmo plano e F1F2 = 2c, temos: Prof: Linhares e Júlio Página 73
  • 74. A figura obtida é uma hipérbole. Observação:Os dois ramos da hipérbole são determinados por um plano paralelo ao eixo de simetria de dois cones circulares retos e opostos pelo vértice: Parábola Dados uma reta d e um ponto F , de um plano , chamamos de parábola o conjunto de pontos do plano eqüidistantes de F e d. Assim, sendo, por exemplo, F, P, Q e R pontos de um plano e d uma reta desse mesmo plano, de modo que nenhum ponto pertença a d, temos: Observações: Prof: Linhares e Júlio Página 74
  • 75. 1ª) A parábola é obtida seccionando-se obliquamente um cone circular reto: 2ª) Os telescópios refletores mais simples têm espelhos com secções planas parabólicas. 3ª) As trajetórias de alguns cometas são parábolas, sendo que o Sol ocupa o foco. 4ª) A superfície de um líquido contido em um cilindro que gira em torno de seu eixo com velocidade constante é parabólica. Prof: Linhares e Júlio Página 75
  • 76. Matrizes Introdução O crescente uso dos computadores tem feito com que a teoria das matrizes seja cada vez mais aplicada em áreas como Economia, Engenharia, Matemática, Física, dentre outras. Vejamos um exemplo. A tabela a seguir representa as notas de três alunos em uma etapa: Química Inglês Literatura Espanhol A 8 7 9 8 B 6 6 7 6 C 4 8 5 9 Se quisermos saber a nota do aluno B em Literatura, basta procurar o número que fica na segunda linha e na terceira coluna da tabela. Vamos agora considerar uma tabela de números dispostos em linhas e colunas, como no exemplo acima, mas colocados entre parênteses ou colchetes: Em tabelas assim dispostas, os números são os elementos. As linhas são enumeradas de cima para baixo e as colunas, da esquerda para direita: Prof: Linhares e Júlio Página 76
  • 77. Tabelas com m linhas e n colunas ( m e n números naturais diferentes de 0) são denominadas matrizes m x n. Na tabela anterior temos, portanto, uma matriz 3 x 3. Veja mais alguns exemplos: é uma matriz do tipo 2 x 3 é uma matriz do tipo 2 x 2 Notação geral Costuma-se representar as matrizes por letras maiúsculas e seus elementos por letras minúsculas, acompanhadas por dois índices que indicam, respectivamente, a linha e a coluna que o elemento ocupa. Assim, uma matriz A do tipo m x n é representada por: ou, abreviadamente, A = [aij]m x n, em que i e j representam, respectivamente, a linha e a coluna que o elemento ocupa. Por exemplo, na matriz anterior, a23 é o elemento da 2ª linha e da 3ª coluna. Prof: Linhares e Júlio Página 77
  • 78. Na matriz , temos: Ou na matriz B = [ -1 0 2 5 ], temos: a11 = -1, a12 = 0, a13 = 2 e a14 = 5. Denominações especiais Algumas matrizes, por suas características, recebem denominações especiais. Matriz linha: matriz do tipo 1 x n, ou seja, com uma única linha. Por exemplo, a matriz A =[4 7 -3 1], do tipo 1 x 4. Matriz coluna: matriz do tipo m x 1, ou seja, com uma única coluna. Por exemplo, , do tipo 3 x 1 Matriz quadrada: matriz do tipo n x n, ou seja, com o mesmo número de linhas e colunas; dizemos que a matriz é de ordem n. Por exemplo, a matriz é do tipo 2 x 2, isto é, quadrada de ordem 2. Numa matriz quadrada definimos a diagonal principal e a diagonal secundária. A principal é formada pelos elementos aij tais que i = j. Na secundária, temos i + j = n + 1. Veja: Prof: Linhares e Júlio Página 78
  • 79. Observe a matriz a seguir: a11 = -1 é elemento da diagonal principal, pis i = j = 1 a31= 5 é elemento da diagonal secundária, pois i + j = n + 1 ( 3 + 1 = 3 + 1) Matriz nula: matriz em que todos os elementos são nulos; é representada por 0m x n. Por exemplo, . Matriz diagonal: matriz quadrada em que todos os elementos que não estão na diagonal principal são nulos. Por exemplo: Matriz identidade: matriz quadrada em que todos os elementos da diagonal principal são iguais a 1 e os demais são nulos; é representada por In, sendo n a ordem da matriz. Por exemplo: Prof: Linhares e Júlio Página 79
  • 80. Assim, para uma matriz identidade . Matriz transposta: matriz At obtida a partir da matriz A trocando- se ordenadamente as linhas por colunas ou as colunas por linhas. Por exemplo: Desse modo, se a matriz A é do tipo m x n, At é do tipo n x m. Note que a 1ª linha de A corresponde à 1ª coluna de At e a 2ª linha de A corresponde à 2ª coluna de At. Matriz simétrica: matriz quadrada de ordem n tal que A = At . Por exemplo, é simétrica, pois a12 = a21 = 5, a13 = a31 = 6, a23 = a32 = 4, ou seja, temos sempre a ij = a ij. Matriz oposta: matriz -A obtida a partir de A trocando-se o sinal de todos os elementos de A. Por exemplo, . Prof: Linhares e Júlio Página 80
  • 81. Igualdade de matrizes Duas matrizes, A e B, do mesmo tipo m x n, são iguais se, e somente se, todos os elementos que ocupam a mesma posição são iguais: . Operações envolvendo matrizes Adição Dadas as matrizes , chamamos de soma dessas matrizes a matriz , tal que Cij = aij + bij , para todo : A+B=C Exemplos: Observação: A + B existe se, e somente se, A e B forem do mesmo tipo. Propriedades Sendo A, B e C matrizes do mesmo tipo ( m x n), temos as seguintes propriedades para a adição: a) comutativa: A + B = B + A b) associativa: ( A + B) + C = A + ( B + C) Prof: Linhares e Júlio Página 81
  • 82. c) elemento neutro: A + 0 = 0 + A = A, sendo 0 a matriz nula m x n d) elemento oposto: A + ( - A) = (-A) + A = 0 Subtração Dadas as matrizes , chamamos de diferença entre essas matrizes a soma de A com a matriz oposta de B: A-B=A+(-B) Observe: Multiplicação de um número real por uma matriz Dados um número real x e uma matriz A do tipo m x n, o produto de x por A é uma matriz B do tipo m x n obtida pela multiplicação de cada elemento de A por x, ou seja, bij = xaij: B = x.A Observe o seguinte exemplo: Propriedades Sendo A e B matrizes do mesmo tipo ( m x n) e x e y números reais quaisquer, valem as seguintes propriedades: a) associativa: x . (yA) = (xy) . A Prof: Linhares e Júlio Página 82
  • 83. b) distributiva de um número real em relação à adição de matrizes: x . (A + B) = xA + xB c) distributiva de uma matriz em relação à adição de dois números reais: (x + y) . A = xA + yA d) elemento neutro : xA = A, para x=1, ou seja, A=A Multiplicação de matrizes O produto de uma matriz por outra não é determinado por meio do produto dos sus respectivos elementos. Assim, o produto das matrizes A = ( aij) m x p e B = ( bij) p x n é a matriz C = (cij) m x n em que cada elemento cij é obtido por meio da soma dos produtos dos elementos correspondentes da i-ésima linha de A pelos elementos da j-ésima coluna B. Vamos multiplicar a matriz para entender como se obtém cada Cij: 1ª linha e 1ª coluna 1ª linha e 2ª coluna 2ª linha e 1ª coluna 2ª linha e 2ª coluna Prof: Linhares e Júlio Página 83
  • 84. Assim, . Observe que: Portanto, .A, ou seja, para a multiplicação de matrizes não vale a propriedade comutativa. Vejamos outro exemplo com as matrizes : Da definição, temos que a matriz produto A . B só existe se o número de colunas de A for igual ao número de linhas de B: Prof: Linhares e Júlio Página 84
  • 85. A matriz produto terá o número de linhas de A (m) e o número de colunas de B(n): Se A3 x 2 e B 2 x 5 , então ( A . B ) 3 x 5 Se A 4 x 1 e B 2 x 3, então não existe o produto Se A 4 x 2 e B 2 x 1, então ( A . B ) 4 x 1 Propriedades Verificadas as condições de existência para a multiplicação de matrizes, valem as seguintes propriedades: a) associativa: ( A . B) . C = A . ( B . C ) b) distributiva em relação à adição: A . ( B + C ) = A . B + A . C ou (A+B).C=A.C+B.C c) elemento neutro: A . In = In . A = A, sendo In a matriz identidade de ordem n Vimos que a propriedade comutativa, geralmente, não vale para a multiplicação de matrizes. Não vale também o anulamento do produto, ou seja: sendo 0 m x n uma matriz nula, A .B =0 m x n não implica, necessariamente, que A = 0 m x n ou B = 0 m x n. Matriz inversa Dada uma matriz A, quadrada, de ordem n, se existir uma matriz A', de mesma ordem, tal que A . A' = A' . A = In , então A' é matriz inversa de A . Representamos a matriz inversa por A-1 . Prof: Linhares e Júlio Página 85
  • 86. Grandezas - Introdução Entendemos por grandeza tudo aquilo que pode ser medido, contado. As grandezas podem ter suas medidas aumentadas ou diminuídas. Alguns exemplos de grandeza: o volume, a massa, a superfície, o comprimento, a capacidade, a velocidade, o tempo, o custo e a produção. É comum ao nosso dia-a-dia situações em que relacionamos duas ou mais grandezas. Por exemplo: Em uma corrida de "quilômetros contra o relógio", quanto maior for a velocidade, menor será o tempo gasto nessa prova. Aqui as grandezas são a velocidade e o tempo. Num forno utilizado para a produção de ferro fundido comum, quanto maior for o tempo de uso, maior será a produção de ferro. Nesse caso, as grandezas são o tempo e a produção. Grandezas diretamente proporcionais Um forno tem sua produção de ferro fundido de acordo com a tabela abaixo: Tempo Produção (Kg) (minutos) 5 100 10 200 15 300 20 400 Observe que uma grandeza varia de acordo com a outra. Essas grandezas são variáveis dependentes. Observe que: Quando duplicamos o tempo, a produção também duplica. 5min ----> 100Kg 10 min ----> 200Kg Quando triplicamos o tempo, a produção também triplica. 5min ----> 100Kg 15 min ----> 300Kg Assim: Prof: Linhares e Júlio Página 86
  • 87. Duas grandezas variáveis dependentes são diretamente proporcionais quando a razão entre os valores da 1ª grandeza é igual a razão entre os valores correspondentes da 2ª Verifique na tabela que a razão entre dois valores de uma grandeza é igual a razão entre os dois valores correspondentes da outra grandeza. Grandezas inversamente proporcionais Um ciclista faz um treino para a prova de "1000 metros contra o relógio", mantendo em cada volta uma velocidade constante e obtendo, assim, um tempo correspondente, conforme a tabela abaixo Velocidade (m/s) Tempo (s) 5 200 8 125 10 100 16 62,5 20 50 Observe que uma grandeza varia de acordo com a outra. Essas grandezas são variáveis dependentes. Observe que: Quando duplicamos a velocidade, o tempo fica reduzido à metade. 5m/s ----> 200s 10 m/s ----> 100s Quando quadriplicamos a velocidade, o tempo fica reduzido à quarta parte. 5m/s ----> 200s 20 m/s ----> 50s Assim: Prof: Linhares e Júlio Página 87
  • 88. Duas grandezas variáveis dependentes são inversamente proporcionais quando a razão entre os valores da 1ª grandeza é igual ao inverso da razão entre os valores correspondentes da 2ª. Verifique na tabela que a razão entre dois valores de uma grandeza é igual ao inverso da razão entre os dois valores correspondentes da outra grandeza. Prof: Linhares e Júlio Página 88
  • 89. POLINÔMIOS  Definição Uma função polinomial ou simplesmente polinômio, é toda função definida pela relação P(x)=anxn + an-1.xn-1 + an-2.xn-2 + ... + a2x2 + a1x + a0. Onde: an, an-1, an-2, ..., a2, a1, a0 são números reais chamados coeficientes. n  IN x  C (nos complexos) é a variável. GRAU DE UM POLINÔMIO: Grau de um polinômio é o expoente máximo que ele possui. Se o coeficiente an0, então o expoente máximo n é dito grau do polinômio e indicamos gr(P)=n. Exemplos: a) P(x)=5 ou P(x)=5.x0 é um polinômio constante, ou seja, gr(P)=0. b) P(x)=3x+5 é um polinômio do 1º grau, isto é, gr(P)=1. c) P(x)=4x5+7x4 é um polinômio do 5º grau, ou seja, gr(P)=5. Obs: Se P(x)=0, não se define o grau do polinômio.  Valor numérico O valor numérico de um polinômio P(x) para x=a, é o número que se obtém substituindo x por a e efetuando todas as operações indicadas pela relação que define o polinômio. Exemplo: Se P(x)=x3+2x2+x-4, o valor numérico de P(x), para x=2, é: P(x)= x3+2x2+x-4 P(2)= 23+2.22+2-4 P(2)= 14 Observação: Se P(a)=0, o número a chamado raiz ou zero de P(x). Por exemplo, no polinômio P(x)=x2-3x+2 temos P(1)=0; logo, 1 é raiz ou zero desse polinômio. Prof: Linhares e Júlio Página 89
  • 90. Alguns exercícios resolvidos: 1º) Sabendo-se que –3 é raiz de P(x)=x3+4x2-ax+1, calcular o valor de a. Resolução: Se –3 é raiz de P(x), então P(-3)=0. P(-3)=0 => (-3)3+4(-3)2-a.(-3)+1 = 0 3a = -10 => a=-10/3 Resposta: a=-10/3 2º) Calcular m  IR para que o polinômio P(x)=(m2-1)x3+(m+1)x2-x+4 seja: a) do 3ºgrau b) do 2º grau c) do 1º grau Resposta: a) para o polinômio ser do 3º grau, os coeficientes de x2 e x3 devem ser diferentes de zero. Então: m2-10 => m21 => m1 m+10 => m-1 Portanto, o polinômio é do 3º grau se m1 e m-1. b) para o polinômio ser do 2º grau, o coeficiente de x 3 deve ser igual a zero e o coeficiente de x2 diferente de zero. Então: m2-1=0 => m2=1 => m=1 m+10 => m-1 Portanto, o polinômio é do 2º grau se m=1. c) para o polinômio ser do 1º grau, os coeficientes de x2 e x3 devem ser iguais a zero. Então: m2-1=0 => m2=1 => m=1 m+1=0 => m=-1 Portanto, o polinômio é do 1º grau se m=-1. 3º) Num polinômio P(x), do 3º grau, o coeficiente de x3 é 1. Se P(1)=P(2)=0 e P(3)=30, calcule o valor de P(-1). Resolução: Temos o polinômio: P(x)=x3+ax2+bx+c. Precisamos encontrar os valores de a,b e c (coeficientes). Vamos utilizar os dados fornecidos pelo enunciado do problema: P(1)=0 => (1)3+a.(1)2+b(1)+c = 0 => 1+a+b+c=0 => a+b+c=-1 P(2)=0 => (2)3+a.(2)2+b(2)+c = 0 => 8+4a+2b+c=0 => 4a+2b+c=-8 P(3)=30 => (3)3+a.(3)2+b(3)+c = 30 => 27+9a+3b+c=30 => 9a+3b+c=3 Prof: Linhares e Júlio Página 90
  • 91. Temos um sistema de três variáveis: a  b  c  -1  4a  2b  c  -8 9a  3b  c  3  Resolvendo esse sistema encontramos as soluções: a=9, b=-34, c=24 Portanto o polinômio em questão é P(x)= x3+9x2-34x+24. O problema pede P(-1): P(-1)= (-1)3+9(-1)2-34(-1)+24 => P(-1)=-1+9+34+24 P(-1)= 66 Resposta: P(-1)= 66  Polinômios iguais Dizemos que dois polinômios A(x) e B(x) são iguais ou idênticos (e indicamos A(x)B(x)) quando assumem valores numéricos iguais para qualquer valor comum atribuído à variável x. A condição para que dois polinômios sejam iguais ou idênticos é que os coeficientes dos termos correspondentes sejam iguais. Exemplo: Calcular a,b e c, sabendo-se que x2-2x+1  a(x2+x+1)+(bx+c)(x+1). Resolução: Eliminando os parênteses e somando os termos semelhantes do segundo membro temos: x2-2x+1  ax2+ax+a+bx2+bx+cx+c 1x2-2x+1  (a+b)x2+(a+b+c)x+(a+c) Agora igualamos os coeficientes correspondentes: a  b  1  a  b  c  2 a  c  1  Substituindo a 1ª equação na 2ª: 1+c = -2 => c=-3. Colocando esse valor de c na 3ª equação, temos: a-3=1 => a=4. Colocando esse valor de a na 1ª equação, temos: 4+b=1 => b=-3. Resposta: a=4, b=-3 e c=-3. Prof: Linhares e Júlio Página 91
  • 92. Obs: um polinômio é dito identicamente nulo se tem todos os seus coeficientes nulos.  Divisão de polinômios Sejam dois polinômios P(x) e D(x), com D(x) não nulo. Efetuar a divisão de P por D é determinar dois polinômios Q(x) e R(x), que satisfaçam as duas condições abaixo: 1ª) Q(x).D(x) + R(x) = P(x) 2ª) gr(R) < gr(D) ou R(x)=0 P( x) D( x ) R( x) Q( x) Nessa divisão: P(x) é o dividendo. D(x) é o divisor. Q(x) é o quociente. R(x) é o resto da divisão. Obs: Quando temos R(x)=0 dizemos que a divisão é exata, ou seja, P(x) é divisível por D(x) ou D(x) é divisor de P(x). Se D(x) é divisor de P(x)  R(x)=0 Exemplo: Determinar o quociente de P(x)=x4+x3-7x2+9x-1 por D(x)=x2+3x-2. Resolução: Aplicando o método da chave, temos: x 4  x3  7 x 2  9 x  1 x 2  3x  2  x 4  3x3  2 x 2 x 2  2 x  1  Q( x)  2 x3  5 x 2  9 x  1  2 x3  6 x 2  4 x x2  5x  1  x 2  3x  2 2 x  1  R( x) Prof: Linhares e Júlio Página 92
  • 93. Verificamos que: x 4  -  1  (x 2  3x - 2) (x 2 - 2x  1)  (2x  1)   x 7x  9x    3 2 -   P(x) D(x) Q(x) R(x)  Divisão de um polinômio por um binômio da forma ax+b Vamos calcular o resto da divisão de P(x)=4x2-2x+3 por D(x)=2x-1. Utilizando o método da chave temos: 4x2  2x  3 2x  1  4x2  2x 2x 3 Logo: R(x)=3 A raiz do divisor é 2x-1=0 => x=1/2. Agora calculamos P(x) para x=1/2. P(1/2) = 4(1/4) – 2(1/2) + 3 P(1/2) = 3 Observe que R(x) = 3 = P(1/2) Portanto, mostramos que o resto da divisão de P(x) por D(x) é igual ao valor numérico de P(x) para x=1/2, isto é, a raiz do divisor.  Teorema do resto O resto da divisão de um polinômio P(x) pelo binômio ax+b é igual a P(-b/a). Note que –b/a é a raiz do divisor. Exemplo: Calcule o resto da divisão de x2+5x-1 por x+1. Resolução: Achamos a raiz do divisor: x+1=0 => x=-1 Pelo teorema do resto sabemos que o resto é igual a P(-1): P(-1)=(-1)2+5.(-1)-1 => P(-1) = -5 = R(x) Resposta: R(x) = -5. Prof: Linhares e Júlio Página 93
  • 94.  Teorema de D’Alembert Um polinômio P(x) é divisível pelo binômio ax+b se P(-b/a)=0 Exemplo: Determinar o valor de p, para que o polinômio P(x)=2x3+5x2- px+2 seja divisível por x-2. Resolução: Se P(x) é divisível por x-2, então P(2)=0. P(2)=0 => 2.8+5.4-2p+2=0 => 16+20-2p+2=0 => p=19 Resposta: p=19.  Divisão de um polinômio pelo produto (x-a)(x-b) Vamos resolver o seguinte problema: calcular o resto da divisão do polinômio P(x) pelo produto (x-a)(x-b), sabendo-se que os restos da divisão de P(x) por (x-a) e por (x-b) são, respectivamente, r1 e r2. Temos: a é a raiz do divisor x-a, portanto P(a)=r1 (eq. 1) b é a raiz do divisor x-b, portanto P(b)=r2 (eq. 2) E para o divisor (x-a)(x-b) temos P(x)=(x-a)(x-b) Q(x) + R(x) (eq. 3) O resto da divisão de P(x) por (x-a)(x-b) é no máximo do 1º grau, pois o divisor é do 2º grau; logo: R(x)=cx+d Da eq.3 vem: P(x)=(x-a)(x-b) Q(x) + cx + d Fazendo: x=a => P(a) = c(a)+d (eq. 4) x=b => P(b) = c(b)+d (eq. 5) Das equações 1, 2, 4 e 5 temos: ca  d  r1  cb  d  r2 Resolvendo o sistema obtemos: Prof: Linhares e Júlio Página 94
  • 95. r1  r2 ar  ar1 c e d 2 , com a  b ab ab r r ar  ar1 Logo : R( x)  1 2 x  2 , com a  b ab ab Observações: 1ª) Se P(x) for divisível por (x-a) e por (x-b), temos: P(a)= r1 =0 P(b)= r2 =0 Portanto, P(x) é divisível pelo produto (x-a)(x-b), pois: r1  r2 ar  ar1 R( x)  x 2  00  0 a b a b 2ª) Generalizando, temos: Se P(x) é divisível por n fatores distintos (x-a1), (x-a2),..., (x-an) então P(x) é divisível pelo produto (x-a1)(x-a2)...(x-an). Exemplo: Um polinômio P(x) dividido por x dá resto 6 e dividido por (x-1) dá resto 8. Qual o resto da divisão de P(x) por x(x-1)? Resolução: 0 é a raiz do divisor x, portanto P(0)=6 (eq. 1) 1 é a raiz do divisor x-1, portanto P(1)=8 (eq. 2) E para o divisor x(x-1) temos P(x)=x(x-1) Q(x) + R(x) (eq. 3) O resto da divisão de P(x) por x(x-1) é no máximo do 1º grau, pois o divisor é do 2º grau; logo: R(x)=ax+b Da eq.3 vem: P(x)=x(x-1) Q(x) + ax + b Fazendo: x=0 => P(0) = a(0)+b => P(0) = b (eq. 4) x=1 => P(1) = a(1)+b => P(1) = a+b (eq. 5) Das equações 1, 2, 4 e 5 temos: b  6  a  b  8 Prof: Linhares e Júlio Página 95
  • 96. Logo, b=6 e a=2. Agora achamos o resto: R(x) = ax+b = 2x+6 Resposta: R(x) = 2x+6.  O dispositivo de Briot-Ruffini Serve para efetuar a divisão de um polinômio P(x) por um binômio da forma (ax+b). Exemplo: Determinar o quociente e o resto da divisão do polinômio P(x)=3x3-5x2+x-2 por (x-2). Resolução:    RAIZ DO DIVISOR ES DE P(x)  COEFICIENT    2 3 5 1 2  3.(2)  5 1.(2)  1 3.(2)  2 1  3  3  4  COEFICIENTES DO QUOCIENTE Q(x) RESTO Observe que o grau de Q(x) é uma unidade inferior ao de P(x), pois o divisor é de grau 1. Resposta: Q(x)=3x2+x+3 e R(x)=4. Para a resolução desse problema seguimos os seguintes passos: 1º) Colocamos a raiz do divisor e os coeficientes do dividendo ordenadamente na parte de cima da “cerquinha”. 2º) O primeiro coeficiente do dividendo é repetido abaixo. 3º) Multiplicamos a raiz do divisor por esse coeficiente repetido abaixo e somamos o produto com o 2º coeficiente do dividendo, colocando o resultado abaixo deste. 4º) Multiplicamos a raiz do divisor pelo número colocado abaixo do 2º coeficiente e somamos o produto com o 3º coeficiente, colocando o resultado abaixo deste, e assim sucessivamente. 5º) Separamos o último número formado, que é igual ao resto da divisão, e os números que ficam à esquerda deste serão os coeficientes do quociente. Prof: Linhares e Júlio Página 96
  • 97.  Decomposição de um polinômio em fatores Vamos analisar dois casos: 1º caso: O polinômio é do 2º grau. De uma forma geral, o polinômio de 2º grau P(x)=ax2+bx+c que admite as raízes r1 e r2 pode ser decomposto em fatores do 1º grau, da seguinte forma: ax2+bx+c = a(x-r1)(x-r2) Exemplos: 1) Fatorar o polinômio P(x)=x2-4. Resolução: Fazendo x2-4=0, obtemos as raízes r1=2 e r2=-2. Logo: x2-4 = (x-2)(x+2). 2) Fatorar o polinômio P(x)=x2-7x+10. Resolução: Fazendo x2-7x+10=0, obtemos as raízes r1=5 e r2=2. Logo: x2-7x+10 = (x-5)(x-2). 2º caso: O polinômio é de grau maior ou igual a 3. Conhecendo uma das raízes de um polinômio de 3º grau, podemos decompô-lo num produto de um polinômio do 1º grau por um polinômio do 2º grau e, se este tiver raízes, podemos em seguida decompô-lo também. Exemplo: Decompor em fatores do 1º grau o polinômio 2x3-x2-x. Resolução: 2x3-x2-x = x.(2x2-x-1)  colocando x em evidência Fazendo x.(2x2-x-1) = 0 obtemos: x=0 ou 2x2-x-1=0. Uma das raízes já encontramos (x=0). As outras duas saem da equação: 2x2-x-1=0 => r1=1 e r2=-1/2. Portanto, o polinômio 2x3-x2-x, na forma fatorada é: 2.x.(x-1).(x+(1/2)). Generalizando, se o polinômio P(x)=anxn+an-1xn-1+...+a1x+a0 admite n raízes r1, r2,..., rn, podemos decompô-lo em fatores da seguinte forma: anxn+an-1xn-1+...+a1x+a0 = an(x-r1)(x-r2)...(x-rn) Prof: Linhares e Júlio Página 97
  • 98. Observações: 1) Se duas, três ou mais raiz forem iguais, dizemos que são raízes duplas, triplas, etc. 2) Uma raiz r1 do polinômio P(x) é dita raiz dupla ou de multiplicidade 2 se P(x) é divisível por (x-r1)2 e não por (x-r1)3. Prof: Linhares e Júlio Página 98
  • 99. PROBABILIDADE A história da teoria das probabilidades, teve início com os jogos de cartas, dados e de roleta. Esse é o motivo da grande existência de exemplos de jogos de azar no estudo da probabilidade. A teoria da probabilidade permite que se calcule a chance de ocorrência de um número em um experimento aleatório. Experimento Aleatório É aquele experimento que quando repetido em iguais condições, podem fornecer resultados diferentes, ou seja, são resultados explicados ao acaso. Quando se fala de tempo e possibilidades de ganho na loteria, a abordagem envolve cálculo de experimento aleatório. Espaço Amostral É o conjunto de todos os resultados possíveis de um experimento aleatório. A letra que representa o espaço amostral, é S. Exemplo: Lançando uma moeda e um dado, simultaneamente, sendo S o espaço amostral, constituído pelos 12 elementos: S = {K1, K2, K3, K4, K5, K6, R1, R2, R3, R4, R5, R6} 1. Escreva explicitamente os seguintes eventos: A={caras e m número par aparece}, B={um número primo aparece}, C={coroas e um número ímpar aparecem}. 2. Idem, o evento em que: a) A ou B ocorrem; b) B e C ocorrem; c) Somente B ocorre. 3. Quais dos eventos A,B e C são mutuamente exclusivos Resolução: 1. Para obter A, escolhemos os elementos de S constituídos de um K e um número par: A={K2, K4, K6}; Prof: Linhares e Júlio Página 99
  • 100. Para obter B, escolhemos os pontos de S constituídos de números primos: B={K2,K3,K5,R2,R3,R5} Para obter C, escolhemos os pontos de S constituídos de um R e um número ímpar: C={R1,R3,R5}. 2. (a) A ou B = AUB = {K2,K4,K6,K3,K5,R2,R3,R5} (b) B e C = B  C = {R3,R5} (c) Escolhemos os elementos de B que não estão em A ou C; B  Ac  Cc = {K3,K5,R2} 3. A e C são mutuamente exclusivos, porque A  C =   Conceito de probabilidade Se em um fenômeno aleatório as possibilidades são igualmente prováveis, então a probabilidade de ocorrer um evento A é: Por, exemplo, no lançamento de um dado, um número par pode ocorrer de 3 maneiras diferentes dentre 6 igualmente prováveis, portanto, P = 3/6= 1/2 = 50% Dizemos que um espaço amostral S (finito) é equiprovável quando seus eventos elementares têm probabilidades iguais de ocorrência. Num espaço amostral equiprovável S (finito), a probabilidade de ocorrência de um evento A é sempre: Prof: Linhares e Júlio Página 100
  • 101. Propriedades Importantes: 1. Se A e A’ são eventos complementares, então: P( A ) + P( A' ) = 1 2. A probabilidade de um evento é sempre um número entre  (probabilidade de evento impossível) e 1 (probabilidade do evento certo). Probabilidade Condicional Antes da realização de um experimento, é necessário que já tenha alguma informação sobre o evento que se deseja observar. Nesse caso, o espaço amostral se modifica e o evento tem a sua probabilidade de ocorrência alterada. Fórmula de Probabilidade Condicional P(E1 e E2 e E3 e ...e En-1 e En) é igual a P(E1).P(E2/E1).P(E3/E1 e E2)...P(En/E1 e E2 e ...En-1). Onde P(E2/E1) é a probabilidade de ocorrer E2, condicionada pelo fato de já ter ocorrido E1; P(E3/E1 e E2) é a probabilidade ocorrer E3, condicionada pelo fato de já terem ocorrido E1 e E2; P(Pn/E1 e E2 e ...En-1) é a probabilidade de ocorrer En, condicionada ao fato de já ter ocorrido E1 e E2...En-1. Exemplo: Uma urna tem 30 bolas, sendo 10 vermelhas e 20 azuis. Se ocorrer um sorteio de 2 bolas, uma de cada vez e sem reposição, qual será a probabilidade de a primeira ser vermelha e a segunda ser azul? Resolução: Seja o espaço amostral S=30 bolas, bolinhas e considerarmos os seguintes eventos: Prof: Linhares e Júlio Página 101